101
RESIDENT REVIEW COURSE STUDY QUESTIONS CLINICAL PATHOLOGY APRIL 20-22, 2006

CP RIse Review

Embed Size (px)

Citation preview

Page 1: CP RIse Review

RESIDENT REVIEW COURSE

STUDY QUESTIONS

CLINICAL PATHOLOGY

APRIL 20-22, 2006

Page 2: CP RIse Review
Page 3: CP RIse Review

Coagulation Questions – Michael Laposata

1. DDAVP generates an increase in the amount of plasma von Willebrand factor primarily by: a) increasing von Willebrand factor synthesis by the endothelium b) increasing von Willebrand factor synthesis by the megakaryocytes c) increasing secretion of von Willebrand factor from endothelium d) increasing secretion of von Willebrand factor from megakaryocytes 2. Which of the following is the mechanism of action of the drug warfarin (Coumadin)? a) interruption of coagulation factor breakdown b) interruption of factor synthesis for factors requiring Vitamin K for production c) inhibition of platelet function d) increase in the activity of fibrinolysis 3. Which molecule is defective in activated protein C resistance? a) protein C b) factor V c) protein S d) thrombomodulin 4. Which of the following is a standard battery of tests used in the initial evaluation of a

patient for von Willebrand's disease? a) bleeding time, platelet aggregation, PTT b) ristocetin co-factor, von Willebrand factor antigen, factor VIII c) factor VIII, factor IX, and PTT d) multimer analysis, crossed immunoelectrophoresis, and bleeding time 5. To which of the following proteins does heparin bind in exerting its anticoagulant effect? a) antithrombin III b) protein C c) protein S d) alpha-2 macroglobulin 6. Which of the following is a site of synthesis of von Willebrand factor? a) endothelial cells b) red blood cells c) white blood cells d) macrophages

Page 4: CP RIse Review

7. Which of the following is a common complication of heparin therapy? a) increased fibrinolysis b) thrombocytopenia c) decreased factor VIII activity d) peripheral vasodilation 8. Which of the following can be used to treat an overdose of heparin associated with

bleeding? a) fresh frozen plasma b) protamine sulfate c) factor VIII concentrate d) epsilon aminocaproic acid 9. A lupus anticoagulant is: a) a proteolytic enzyme b) a small molecular weight lipid c) a long chain proteoglycan d) an immunoglobulin 10. DIC is a disorder more commonly associated with: a) bleeding b) thrombosis c) neither bleeding nor thrombosis d) slight bleeding with major arterial thrombosis 11. Which of the following are four commonly used tests for the diagnosis of DIC? a) FDP or D-Dimer, fibrinogen, PT, platelet count b) PT, PTT, thrombin time, reptilase time c) FDP or D-Dimer, Factor V, Factor VIII, antithrombin III d) protein C, protein S, antithrombin III, lupus inhibitor 12. Which of the following is a common predisposing condition for DIC? a) rheumatoid arthritis b) severe gram negative infection c) ulcer of the stomach d) allergy 13. In heparin-induced thrombocytopenia:

a) there is a high risk for thrombosis b) heparin therapy should be maintained in the presence of severe thrombocytopenia c) limited heparin exposure is nearly always safe d) low molecular weight heparin is a commonly effective therapy

Page 5: CP RIse Review

14. Which of the following types of von Willebrand's disease is the most common? a) type 1 b) type 2 c) type 3 d) acquired von Willebrand's disease 15. Which factor deficiency is the most common of the choices below? a) factor X deficiency b) factor VIII deficiency c) factor V deficiency d) factor II deficiency 16. Protein C deficiency is which of the following? a) a bleeding disorder b) a thrombotic disorder c) both d) neither 17. The name of the clinical condition characterized by a blood clot in the lung is: a) deep venous thrombosis b) pulmonary embolism c) purpura d) petechiae 18. This patient has a factor VIII level of 8%. His hemophilia would be classified as? a) severe b) moderate c) mild d) asymptomatic 19. Which of the following disorders is inherited in a sex linked transmission? a) von Willebrand's disease b) hemophilia B c) protein C deficiency d) plasminogen deficiency 20. Which one of the following factor deficiencies is not associated with clinical bleeding, even if there is no factor (0%) present? a) factor II b) factor V c) factor IX d) factor XII

Page 6: CP RIse Review

21. Which of the following statements is true? a) protein S has proteolytic activity b) thrombomodulin is a circulating plasma protein

c) the thrombin-thrombomodulin complex converts protein C to activated protein C d) protein S bound to C4b binding protein is active as a protein C cofactor 22. Which of the following deficiencies or condition is not a predisposing factor for

thrombosis? a) protein C deficiency b) activated protein C resistance c) protein S deficiency d) factor XIII deficiency 23. Which of the following parameters is the most widely accepted monitor for coumadin therapy? a) the PT value in seconds b) the PT ratio of patient PT/control PT c) the INR (international normalized ratio) d) the PTT value in seconds 24. A woman who is a carrier for hemophilia has 4 daughters and 4 sons. Which of the

following statements is true? a) all of the boys should be hemophiliacs b) half of the boys should be hemophiliacs c) all of the girls should be obligate carriers of the hemophilia gene d) none of the girls should be carriers of the hemophilia gene 25. When patients must be treated with anticoagulants for pulmonary embolism, which of the

following is the most common treatment? a) first heparin or low molecular weight heparin and then warfarin b) first warfarin and then heparin or low molecular weight heparin

c) heparin or low molecular weight heparin only, even after discharge from the hospital d) warfarin only 26. This patient has a normal PT and an elevated PTT that completely corrects in a PTT

mixing study. This patient does not have a clinical history of thrombosis or spontaneous abortions. What would be the next best tests to perform?

a) factors VIII, IX, XI, and XII b) lupus anticoagulant assays c) factor VII assay d) platelet aggregation studies

Page 7: CP RIse Review

27. Which of the following are the two most common congenital bleeding disorders? a) factor VIII deficiency and factor II deficiency b) plasminogen deficiency and protein C deficiency c) von Willebrand's disease and hemophilia A d) hemophilia A and hemophilia B 28. Which of the following is the most common congenital cause for hypercoagulability? a) protein C deficiency b) protein S deficiency c) antithrombin III deficiency d) activated protein C resistance 29. This patient has a prolonged PTT which is 65 seconds. In the mixing study, the PTT

immediately upon mixing is 35 seconds and at 60 minutes of incubation, the PTT is 55 seconds. This is most likely to represent:

a) a lupus inhibitor b) factor XI deficiency c) factor VIII inhibitor

d) plasminogen activator inhibitor 30. Which of the following drugs is commonly implicated in drug-induced

thrombocytopenia?

a) epsilon amino caproic acid b) aspirin c) heparin d) topical thrombin

31. This male patient has a factor VIII level of 30%. There is a history of mild bleeding in the family in which both men and women have been affected. Which of the following statements is most likely to be true?

a) the patient is more likely to have von Willebrand's disease than hemophilia

b) the patient is more likely to have hemophilia than von Willebrand's disease c) the patient is not likely to have either hemophilia or von Willebrand's disease d) there is an equal chance that the patient has hemophilia or von Willebrand's disease 32. A deficiency of factor XI prolongs: a) the PTT b) the PT c) the PT and the PTT d) neither

Page 8: CP RIse Review

33. Which one of the following factors cannot be measured by assay of plasma coagulation factors? a) factor IX b) factor V c) factor VI d) factor II 34. Which one of the following factors is not Vitamin K dependent? a) factor VII b) factor IX c) factor II d) factor V 35. This patient has a PTT of 30 seconds (normal) and a PT of 25 seconds (prolonged).

Which of the following deficiencies is most likely? a) factor IX deficiency b) factor VII deficiency c) factor XII deficiency d) factor II deficiency 36. Which of the following is true regarding thrombomodulin? a) it binds protein S and makes it unavailable to serve as a cofactor for protein C b) inhibits activated protein C c) it is a cofactor for protein C in the inactivation of factors Va and VIIIa

d) it binds thrombin, and the thrombin-thrombomodulin complex activates protein C to its activated form

37. For patients on heparin therapy, the minimum amount of anticoagulation as judged by the

PTT is: a) 2.5 X (mean of normal range) b) 3.0 X (mean of normal range) c) 1.5 X (mean of normal range) d) 1.0 X (mean of normal range) 38. This patient has a hypercoagulation defect and is not responsive to heparin therapy. The

most likely diagnosis is: a) protein C deficiency b) protein S deficiency c) antithrombin III deficiency d) plasminogen deficiency

Page 9: CP RIse Review

39. A patient has a factor VIII level of 3%. His hemophilia is classified as: a) severe b) moderate

c) mild d) asymptomatic

40. This patient has a platelet count of 250,000/mL and a normal PT and PTT. The platelet

aggregation studies are abnormal. Which of the following is true?

a) the patient has a qualitative platelet disorder but not a quantitative platelet disorder b) the patient has a quantitative platelet disorder but not a qualitative platelet disorder

c) the patient has both a quantitative and a qualitative platelet disorder d) the patient has no platelet disorder 41. Which of the following enzymes is inhibited by aspirin? a) Vitamin K epoxide reductase b) thromboxane synthetase c) cyclooxygenase d) urokinase 42. Which of the following is the strongest platelet agonist? a) ADP b) epinephrine c) saline d) collagen 43. Which of the following products is pooled from many donors? a) cryoprecipitate b) factor VIII concentrate c) fresh frozen plasma

d) platelet concentrates 44. In the patient treated with heparin whose PTT is greater than 150 seconds and has

significant clinical bleeding, what can the patient be given to prevent the patient from bleeding to death?

a) protamine b) plasma c) vitamin K d) streptokinase

Page 10: CP RIse Review

45. What is the formula for the INR?

a) INR = (PT patient at time 0)ISI

( PT patient at 1 hr)

b) INR = (PT patient )ISI

(PT control )

c) INR = PT patient PT control

d) INR = (PT control)ISI

(PT patient) 46. The major site of synthesis of coagulation factors is the? a) bone marrow b) endothelium c) liver

d) spleen and lymph nodes 47. Among the choices below, which factor is the earliest one involved in the initiation of

coagulation?

a) XI b) II c) VII d) VIII

48. Which factor gene has a mutation that increases its level in most affected patients and

predisposes to hypercoagulability?

a) V b) IX c) X d) II

49. Which factor circulates in a complex with von Willebrand factor?

a) VIII b) IX c) VII d) XI

50. Which factor binds to tissue factor and activates factor IX and factor X?

a) VII b) VIII c) VI d) II

Page 11: CP RIse Review

51. Which of the following drugs or drug class is not a platelet function inhibitor?

a) Coumadin b) Aspirin c) Glycoprotein IIb-IIIa inhibitors d) Ibuprofen (an NSAID)

52. The minimum concentration which should provide normal hemostasis (assuming normal

platelet function) in a patient undergoing surgery is:

a) 5,000/µL b) 500,000/µL c) 1,000/µL d) 100,000/µL

53. Which of the following is not included in a hypercoagulation workup? a) antithrombin III assay b) protein C assay c) activated protein C resistance assay d) ristocetin cofactor assay 54. Which factor deficiency is largely limited to patients of Jewish descent? a) factor XII b) factor XI c) factor X d) factor IX 55. Which of the following is a site or synthesis of von Willebrand factor? a) endothelial cells b) red blood cells c) white blood cells d) macrophages 56. Which clinical sign or symptom in a bleeding patient is most indicative of a

coagulopathy?

a) easy bruising b) gum bleeding with teeth brushing c) relatives with a similar bleeding disorder d) epistaxis with spontaneous stoppage

Page 12: CP RIse Review

57. The platelet aggregation test measures: a) coagulation factor activity b) platelet function c) fibrinolytic activity d) vascular contractility 58. Which blood type is associated with the lowest mean levels of von Willebrand factor? a) type AB b) type A c) type B d) type O 59. Which of the following is not an immune mediated thrombocytopenia?

a) ITP b) post-transfusion purpura c) drug-induced thrombocytopenia d) DIC

60. What is the mean level of factor VIII among female carriers of the hemophilia A gene? a) 30% b) 25% c) 50% d) 2% 61. Liver disease prolongs the PT and PTT as a result of:

a) increased consumption of coagulation factors b) immune destruction of platelets c) increased production of coagulation factors d) decreased synthesis of coagulation factors

62. In clot formation, which one of the following is true?

a) vessel wall contraction is an early event b) platelet aggregation follows after fibrin strand formation is complete c) coagulation factor pathway activation results in vessel wall contraction d) clot formation is independent of platelet function

63. The antigen in heparin-induced thrombocytopenia is:

a) heparin-platelet factor 4 complex b) platelet factor 4 c) beta-thromboglogulin d) heparin-beta-thromboglobulin complex

Page 13: CP RIse Review

64. A qualitative platelet disorder is best diagnosed by:

a) bleeding time b) PT and PTT c) platelet count d) platelet aggregation assay

65. What is the most common site for formation of a deep venous thrombosis?

a) the lungs b) the legs c) the arms d) the abdomen

66. Which of the following is a critical step in the management of the patient with heparin

induced thrombocytopenia?

a) initiate warfarin b) discontinue heparin c) initiate heparin d) discontinue all anticoagulants

67. What of the following conditions is associated with hemophilia B?

a) Factor VIII deficiency b) Factor IX deficiency c) Factor XI deficiency d) Factor XII deficiency

68. Which of the following conditions is much higher in Caucasians than those of Asian or

African descent?

a) Hyperhomocysteinemia b) Factor V Leiden c) Antithrombin deficiency d) Protein C deficiency

69. Homocysteine is derived from:

a) Arginine b) Lysine c) Methionine d) Threonine

70. The thrombotic predisposition associated with an altered homocysteine level that is:

a) increased b) decreased c) frequently changeing d) low, produced by high folate intake

Page 14: CP RIse Review

71. The prothrombin 20210 mutation presents an increased predisposition to:

a) gum bleeding b) thrombosis c) bruising d) excess bleeding with surgery

72. Fresh frozen plasma (FFP) can be used to treat an overdose of:

a) heparin b) aspirin c) low molecular weight heparin d) warfarin

73. Which Factors are reduced with warfarin (coumadin) therapy?

a) II, V, VII, X b) II, VII, IX, XI c) II, VII, IX, X d) II, VII, VIII, IX

74. For patients on heparin therapy, the most commonly used test to monitor the correct

dosage is the:

a) PT b) Thrombin time c) Reptilase time d) PTT

75. A man has hemophilia, has 4 daughters and 4 sons. Which of the following statements is

true?

a) All of the boys should be hemophiliacs b) Half of the boys should be hemophiliacs c) All of the girls should be obligate carriers of the hemophilia gene d) None of the girls should be carriers of the hemophilia gene

Page 15: CP RIse Review

ANSWERS: Coagulation - Michael Laposata, MD, PhD

1. c 2. b 3. b 4. b 5. a 6. a 7. b 8. b 9. d 10. a 11. a 12. b 13. a 14. a 15. b 16. b 17. b 18. c 19. b 20. d 21. c 22. d 23. c 24. b 25. a 26. a 27. c 28. d 29. c 30. c 31. a 32. a 33. c 34. d 35. b 36. d 37. c 38. c 39. b 40. a 41. c 42. d 43. b 44. a 45. b 46. c 47. c 48. d

49. a 50. a 51. a 52. d 53. d 54. b 55. a 56. c 57. b 58. d 59. d 60. c 61. d 62. a 63. a 64. d 65. b 66. b 67. b 68. b 69. c 70. a 71. b 72. d 73. c 74. d 75. c

Page 16: CP RIse Review
Page 17: CP RIse Review

Microbiology I and II Multiple Choice Questions 1. If a human develops cysticercosis, by which means was the infection transmitted?

(a) Eating beef (b) Eating pork (c) Fecal oral from pig feces (d) Fecal oral from cow feces (e) Fecal oral from human feces

2. Which infection predisposes to bladder cancer?

(a) Schistosoma haematobium (b) Schistosoma mansoni (c) Schistosoma japonicum (d) Clonorchis sinensis

3. A healthy, non-pregnant, college-aged female complains of dysuria and urinary

frequency. A urine dipstick shows 2+ leukocyte esterase, with a negative nitrite. A urine sediment shows 50 WBC and few bacteria. How should a urine culture report of rare (102) Staph. saprophyticus in her urine be interpreted?

(a) There are too few organisms in the urine culture to represent significant bacteruria; the patient does not have a UTI. (b) Taking the findings together, there is adequate evidence of a UTI. (c) The findings are inconclusive. Another urine culture should be sent. (d) The findings are inconclusive. Another urine dipstick should be sent.

4. Neisseria gonorrhoeae is able to ferment which of the following carbohydrates:

(a) Glucose only (b) Glucose and maltose (c) Glucose, maltose, and sucrose (d) Glucose, maltose, sucrose, and lactose

5. Which species of Shigella is most prominent in the US?

(a) S. dysenteriae (serogroup A) (b) S. flexneri (serogroup B) (c) S. boydii (serogroup C) (d) S. sonnei (serogroup D)

6. Which species of Shigella is associated with the hemolytic-uremic syndrome?

(a) S. dysenteriae (serogroup A) (b) S. flexneri (serogroup B) (c) S. boydii (serogroup C) (d) S. sonnei (serogroup D)

Page 18: CP RIse Review

7. Which is true of E. coli O157:H7?

(a) Ferments sorbitol (b) Does not ferment sorbitol (c) Does not ferment glucose (d) Shows fluorescence when grown on MUG-containing media

8. RNA testing in HCV infection can be used to:

(a) Predict response to therapy (b) Assess response to therapy (c) Confirm the presence of HCV infection (d) All of the above

9. Which of the following is a test used to differentiate between Streptococcus pneumoniae

and viridans streptococci?

(a) Bacitracin susceptibility test (b) Optochin susceptibility test (c) Novobiocin susceptibility test (d) Vancomycin susceptibility test

10. Why won’t H. influenzae grow on sheep blood agar?

(a) Sheep blood cells produce neither X factor nor V factor (b) Sheep blood cells produce only X factor (c) Sheep blood cells produce only V factor (d) Sheep blood cells produce both X and V factors, but an enzyme present in the blood

hydrolyzes the V factor 11. A primary gram stain shows filamentous, branching gram positive rods. A modified acid

fast stain from the same specimen reveals that the bacteria are modified acid fast positive. The organism is most likely to be which of the following:

(a) Actinomyces (b) Streptomyces (c) Nocardia (d) Mycobacterium tuberculosis

12. A lung biopsy shows an invasive fungal pneumonia. Morphologically, the organism has

relatively narrow, regularly septated, hyaline hyphae, with acute angle branching. Although you suspect Aspergillus, which of the following cannot be ruled out?

(a) Mucor (b) Coccidioides immitis (c) Pseudallescheria boydii/Scedosporium apiospermum (d) Rhizopus

Page 19: CP RIse Review

13. Which of the following dimorphic fungi produces a diffusible red pigment when grown on standard culture media?

(a) Histoplasma capsulatum (b) Penicillium marneffei (c) Blastomyces dermatitidis (d) Paracoccidiodes brasiliensis

14. A 9 year old child visits her primary care physician with signs and symptoms of pertussis

(whooping cough). The patient’s cough began 20 days prior to this visit. The child’s mother is fairly certain that the patient was properly vaccinated. In order to confirm the diagnosis, the clinician should:

(a) Send a nasopharyngeal aspirate or swab for B. pertussis culture (b) Send blood for B. pertussis serology (c) Send for both culture and serology (d) Do nothing—no workup is indicated

15. When a mycobacterium is described as a scotochromogen, this means that colonies of the

organism on solid media are:

(a) Non-pigmented whether grown in dark or light (b) Pigmented whether grown in dark or light (c) Non-pigmented when grown in the dark, but pigmented after exposure to light (d) Pigmented when grown in the dark, but the pigment disappears after exposure to light

16. All of the following are characteristics of dermatophytes except:

(a) Preference for a keratin-rich environment (b) Cause infection in both healthy and immunocompromised patients (c) Hyphae are darkly pigmented (d) Able to grow in the presence of cycloheximide

17. All of the following are characteristics of rapid growing mycobacteria except:

(a) Positive for arylsulfatase activity in 3 days (b) Non-pigmented (c) Cell wall lacks mycolic acids (d) Growth on MacConkey agar lacking crystal violet

18. Trichomonas vaginalis is an example of which type of protozoa:

(a) Amoeba (b) Flagellate (c) Ciliate (d) Sporozoan

Page 20: CP RIse Review

19. A 20 year old pregnant female is screened for HIV using the ELISA method. The ELISA is positive, so a Western blot is run for confirmation. The Western blot is negative. The correct interpretation of these findings is:

(a) The Western blot result is a false negative. The patient is infected with HIV. (b) The ELISA result is a false positive. The patient is not infected with HIV. (c) The results are indeterminant. The ELISA should be repeated after a few months. (d) The results are indeterminant. The Western blot should be repeated after a few months.

20. An 18 year old male presents to his primary care physician with a sore throat, a fever of 100.5 F, and enlarged submandibular lymph nodes. The clinician realizes that acute EBV infection is in the differential diagnosis, and orders a heterophile antibody test. The test is negative. The correct interpretation of these findings is:

(a) The patient does not have acute EBV. (b) The patient does have acute EBV. (c) The negative test cannot rule out acute EBV infection. The heterophile antibody test

should be repeated after a few weeks. (d) The negative test cannot rule out acute EBV infection. There is no point in repeating

the heterophile antibody test, because if it is negative the first time, it will always be negative.

21. A 65 year old female is recovering from a lung transplant in the hospital. She has a 2- week old central line. She develops a fever, and blood cultures are sent. Three of four blood culture bottles are positive for Candida glabrata. The intern taking care of her suggests treating her with fluconazole. The attending physician should:

(a) Agree with the intern and treat with fluconazole. Fluconazole is nearly always active against Candida glabrata.

(b) Disagree with the intern and treat with another antifungal agent (e.g. Amphotericin B). A significant proportion of Candida glabrata isolates from blood are fluconazole resistant.

(c) Disagree with the intern and treat with another antifungal agent (e.g. Amphotericin B). Candida glabrata is always resistant to fluconazole.

(d) Disagree with the intern. Fluconazole is never used to treat Candidemia. 22. A 54 year old male has a history of a mitral valve replacement 18 months prior to admission. He presented to the emergency room with fever and chills, 2 days after he saw his dentist for a root canal. He says he did not take prophylactic antibiotics prior to the procedure, but did take them the next day. Blood culture are drawn, and 2/4 bottles are positive for gram positive cocci in chains. The blood cultures are subcultured to sheep blood agar with an optochin disk. Growth on the plate shows which alpha- hemolytic colonies that are resistant to optochin. A catalase test is negative. The bile esculin test is positive, but the PYR test is negative, and the isolate cannot grow in the presence of 6.5% NaCl. The correct identification of this organism is:

(a) Streptococcus pyogenes (b) Streptococcus pneumoniae (c) Viridans streptococcus (d) Streptococcus bovis

Page 21: CP RIse Review

Answers: for Microbiology I & II – John Branda, MD 1. E 2. A 3. B 4. A 5. D 6. A 7. B 8. D 9. B 10. D 11. C 12. C 13. B 14. A 15. B 16. C 17. C 18. B 19. D 20. C. 21. B 22. D

Page 22: CP RIse Review
Page 23: CP RIse Review

ASCP Review Course Review Questions: Blood Banking and Transfusion Medicine

The answers to these questions may be found in the handout and in a key at the end of this section.

1. Which potential donor is not eligible?

a. had hepatitis A at age 35. b. traveled to Cameroon 15 months ago and took anti-malarial drugs for

prophylaxis. c. made numerous business trips to Germany for a cumulative total of 4

months between 1989 and 1993. d. has a wife with Creutzfeldt-Jakob Disease.

2. Group A donor red blood cells may be safely transfused to a patient with no unexpected alloantibodies who is group AB because:

a. the patient has no unexpected red cell alloantibodies. b. the patient lacks anti-A. c. the donor unit lacks anti-A. d. the patient lacks anti-B.

3. Which potential donor is not eligible?

a. took ibuprofen yesterday for a backache incurred putting in a slate patio. b. was vaccinated three weeks ago for yellow fever in preparation for a trip

to Brazil next month. c. is taking pseudephedrine and acetaminophen for sinus congestion. d. is using Propecia.

4. Which donor is eligible for whole blood donation?

a. Man with hemoglobin of 12 g/dl by venipuncture. b. Woman with hematocrit of 36% by venipuncture. c. Woman with a hemoglobin of 12 g/dl by venipuncture. d. Man or woman with a hemoglobin of 12.5 g/dl by venipuncture. e. Woman with hematocrit of 37% by earlobe sample.

Page 24: CP RIse Review

5. Which potential autologous donor is not eligible to donate whole blood?

a. donated a unit three days ago. b. is taking prednisone for Systemic Lupus Erythematosus. c. was having sexual relations with an IV drug user until three months ago. d. is on the last day of Bactrim prescribed for an episode of dysuria and

lower abdominal pain two weeks ago 6. Which of the following donors is eligible to donate?

a. A 53 year old woman who was taking Soriatane for psoriasis until 22

montha ago. b. A 45 year old man taking Propecia for male pattern baldness. c. A 46 year old woman who took Tegison for psoriasis about 8 years ago. d. A 23 year old man who was taking Accutane for acne until 3 weeks ago. e. A 67 year old man who was taking Proscar for benign prostatic

hypertrophy but stopped the drug 5 weeks ago.

7. Which of the following statements about donation of platelets by apheresis is incorrect?

a. A donor must have a platelet count of at least 150,000 /µL to be able to

donate without prior approval of the medical director. b. A donor may not donate more frequently than once every 56 days unless

s/he has the approval of the medical director c. A donor may not donate more frequently than 24 times in a year. d. A donor who has given a whole blood donation may not donate platelets

for 8 weeks unless the extracorporeal volume of the apheresis instrument is less than 100 mL.

e. A donor must not have taken aspirin or any medication which contains aspirin within the 36 hours preceding the collection.

8. With respect to autologous whole blood donation, which of the following statements is correct?

a. Unused units may be used for other patients if the autologous donor has

met all of the eligibility requirements. b. The donation must be made 96 hours before the day of surgery. c. The donor must meet the routine requirements for allogeneic donation. d. A 7 day interval between donations is required. e. A donor with a hemoglobin level of 11 g/dl is eligible.

Page 25: CP RIse Review

9. Which storage period and anticoagulant/preservative system are incorrectly

paired?

a. ACD: 21 days. b. CPD: 21 days. c. CPDA: 35 days. d. CP2D: 35 days e. AS: 42 days.

10. The reason for removing glycerol from frozen-thawed RBCs by extensive washing is:

a. Glycerol causes hemolysis. b. Glycerol has not approved by the Food and Drug Administration for

intravenous administration. c. Glycerol can cause anaphylactic reactions. d. Glycerol is nephrotoxic e. Glycerol can cause thrombocytopenia.

11. A mating between which two ABO phenotypes could produce children with any of the common four blood types (A, B, O and AB)?

a. AB and AB b. AB and O. c. AB and A. d. AB and B e. A and B.

12. Which of the following viral test is not required of each unit of donated blood

according to the AABB Standards?

a. HBsAg. b. Anti-HBs. c. Anti-HCV. d. Anti-HIV-1/2. e. Anti-HTLV-I/II.

13. Which statement is true?

a. All secretors are Leb. b. All Lea people are non-secretors. c. All non-secretors are Lea. d. All Leb people are non-secretors.

Page 26: CP RIse Review

14.The results of the antigen typing of a patient's red blood cells are shown. What is the most likely genotype?

Reactions with anti-sera to: Sample D C c E e patient's red cells + - + + +

a. Ror" b. R1r c. R1R2 d. R2r

15.Which of the following phenotypes or are more common in Blacks than in

Whites?

a. Dce (R°). b. Js(a+b+). c. Fy(a-b-). d. Le(a-b-). e. All of the above.

16. A 32 year old Rh(D) negative woman (G2P1) presents for routine antenatal care.

She delivered a healthy baby boy who was Rh(D) positive a year ago. During that pregancy, she received 1 dose of Rh immune globulin at 28 weeks of gestation and the day after the baby was delivered. A rosette test done at the time of delivery was negative. Her antibody screen now demonstrates anti-D + anti-C. She has no history of transfusion. Antigen testing of the patient, her husband, and her one year old reveals:

Your conclusion is:

a. The antibody detected in the serum is Rh immune globulin. b. The anti-C is a naturally occurring antibody c. The patient has probably made anti-G. She should received Rh immune

globulin at 28 weeks of gestation. d. The anti-C reagent is defective and needs to be run again with controls. e. The Rh immune globulin she received for the first pregnancy was

insufficient. Serial anti-D titers should be obtained to monitor this pregnancy.

________________ Typing Antisera____________ _________Anti-D___Anti-C___Anti-E___Anti-c___Anti-e___ Patient - - - + + Father + - - + + Infant + - - + +

Page 27: CP RIse Review

17. A 45-year old female donates a unit of whole blood at a blood drive at her church. She is a first time donor. Her ABO and Rh typing are given in the table below. Her ABO/Rh type is:

a. Group A, Rh negative. b. Group O, Rh positive. c. Group A, weak D. d. Group A, Rh positive. e. Group O, Rh negative.

18. Which set of properties is characteristic of the pathogenic autoantibody of Cold

Paroxysmal Hemoglobinuria?

a. IgG, anti-P, hemolytic in vitro b. IgG, anti-I, hemolytic in vitro c. IgG, anti-P, not hemolytic in vitro d. IgM, anti-I, not hemolytic in vitro

19. Which pre-transfusion test is not required for patients?

a. patient RBC with reagent anti-A, anti-B and anti-A,B b. patient serum/plasma with reagent A1 and B cells c. patient RBC with reagent anti-D d. patient serum/plasma with reagent screening red cells

20. A 45 year old man was the victim of an industrial accident three years ago at

which time he received 47 units of RBCs. When he was admitted though the emergency room, he was found to be A, Rh (D) negative with a negative antibody screen. During his 4 week hospitalization, however, he developed the following alloantibodies: anti-D, -C, -E, -K, -S, and -Jkb. In the past few years, he has donated 3 units of autologous blood which have been stored frozen. He is now admitted with a lower GI bleed and the clinicians have ordered RBC transfusions. However, in the time it took to deglycerolize his autologous units, the patient has been stabilized and his hemoglobin level seems to be holding steady around 9.7 g/dL. A medical director of the blood bank, what should you do?

a. Hold the deglycerolized units for an additional24 hours.

______ Forward Typing_________________ __Reverse Typing Weak D Anti-A Anti-B Anti-A,B Anti-D Anti-D DControl A Cells B Cells

O O O O 2+ O 4+ 4+

Page 28: CP RIse Review

b. Refreeze the units but document the rationale for doing so c. Tell the medical staff to transfuse the units since compatible allogeneic

units will be difficult to find. d. Store the units at 4C until the routine expiration time and discard if not

transfused. e. Hold the units until close to their expiration time and then release to

another patient so they will not be wasted..

21. With respect warm autoimmune hemolytic anemia, all of the following are generally true except:

a. Treatment may include steroids, IVIg, or splenectomy. b. Crossmatches are often incompatible. c. The autoantibody reacts with all screen and panel red cells. d. The DAT is positive with anti- IgG. e. RBC transfusions stimulate hemolysis and should be avoided.

22. Potential complications of massive transfusion include all except:

a. hypothermia b. increased P50 c. hypokalemia d. thrombocytopenia

23. The patient is group A, Lea, non-secretor. Which one of the following is true? Red Cell Antigens Secreted Antigens a. A A, Lea b. A, Lea none c. A, Lea A, Lea d. A, Lea Lea

24. The McLeod phenotype is associated with all of the following except: a. Serum creatine phosphokinase levels are elevated. b. Serum contains anti-K. c. Kell antigens expression on RBC is weak. d. The peripheral blood smear demonstratesmany acanthocytes. e. It may be associated with chronic granulomatous disease (CGD).

Page 29: CP RIse Review

25. Which membrane glycoprotein expresses the Ss blood group antigens?

a. Glycophorin A. b. Glycophorin B. c. Glycophorin C. d. Glycophorin D. e. None of the above.

26. A blood donor of which ethnic background is most likely to lack the U antigen?

a. Native American. b. European. c. Asian. d. African. e. Pacific Islander.

27. Compared to one unit of fresh frozen plasma, one unit of cryoprecipitate has:

a. more Factor VIII and in higher concentration b. less Factor VIII and in higher concentration c. more fibrinogen and in higher concentration d. less Factor XIII and in lower concentration

28. Which characteristic of cold red cell autoantibodies is the most predictive of

hemolysis?

a. titre b. thermal amplitude c. specificity d. clonality

29. "Naturally" occurring antibodies are found to which one of the following groups of

carbohydrate blood group antigens? a. A/B, P1, Lea, Se b. A/B, I, Lea, Se c. A/B, I, Lea, P1 d. A/B, p, Se, Lea

30. For the purpose of drawing samples for pretransfusion testing, which of the

following forms of patient identification would not be adequate?

a. Hospital identification bracelet. b. Blood bank identification bracelet. c. The patient stating and spelling his or her name.

Page 30: CP RIse Review

d. Identification of the patient by a third party if the patient is unresponsive. e. The hospital chart on the foot of the patient’s bed.

31. Which of the following samples is acceptable for pretransfusion testing?

a. A sample labeled at the patient’s bedside with the patient’s full name and

hospital number. b. A sample labeled at the patient’s bedside with the date, patient’s full

name, and the phlebotomist’s initials. c. A sample labeled at the nursing station with the patient’s full name,

hospital number, date and initials of the phlebotomist. d. A sample labeled in the hallway outside the patient’s room with the

patient’s full name, hospital number, the date, and initials of the phlebotomist.

e. A sample labeled at the patient’s bedside with the patient’s full name and the hospital number and the date.

32. Potential complications of intravenous immunogloblin administration include:

a. headache b. renal failure c. urticaria d. agranulocytosis e. all of the above

33. A 43-year-old female patient with multiple admissions for upper GI bleeding and

associated with longstanding alcohol abuse presents to the emergency room vomiting large volumes of bright red blood. She has a heart rate of 110 bpm and a systolic blood pressure of 58 mm Hg. Which one of the following blood is the least appropriate in this situation?

a. Fresh frozen plasma. b. Normal saline. c. 5% albumin. d. Whole blood. e. Red blood cells.

34. Your patient develops urticaria and mild dyspnea while receiving the second of

two units of fresh frozen plasma. You should do all of the following except:

a. discontinue the transfusion b. assess the patient c. order 25 mg diphenhydramine d. send an IgA level

Page 31: CP RIse Review

35. In most Rh negative Caucasian individuals, the RH locus on chromosome 1 consists of:

a. one c gene, one e gene and one d gene b. one RHCE gene, no RHD gene c. one RHCE gene, one RHD gene d. no RHCE gene, no RHD gene

36. Your patient with AML has been receiving platelet transfusions for chemotherapy

induced thrombocytopenia. The morning platelet count has been 3,000 to 5,000/µL for the past four days despite receiving 6 to 12 units of platelet concentrates a day. You should:

a. calculate the CCI from the past few days b. order leukocyte reduced platelets c. calculate the CCI 30 minutes after the next platelet transfusion d. order apheresis platelets

37. The P system differs from all of the other carbohydrate blood group systems in one

of the following characteristics: a. P antigens are not affected by secretor status. b. The most common alloantibody, anti-P1, is usually IgG. c. P antigens are found only on glycolipids. d. P glycolipids are adsorbed from the plasma onto the red blood cell

membrane.

38. All of the following are complications of transfusion in sickle cell disease except:

a. Iron overload. b. Formation of rare antibodies. c. Formation of autoantibodies. d. Formation of multiple antibodies. e. Delayed hemolytic transfusion reactions

39. The most common cause of transfusion-associated mortality reported to the FDA

is: a. Transfusion-associated HCV infection. b. Posttransfusion purpura (PTP). c. Transfusion-related acute lung injury (TRALI). d. Intravascular hemolytic transfusion reaction due to ABO incompatibility. e. Transfusion-associated graft-vs-host disease (TA-GVHD).

Page 32: CP RIse Review

40. Graft-vs-host disease is mediated principally by which cell type?

a. B cells. b. T cells. c. Monocytes. d. NK cells. e. Progenitor cells.

41. A patient undergoing plasmapheresis for acute myesthenic crisis develops nausea

and tingling lips. The blood pressure is stable. The patient’s symptoms would probably be relieved by:

a. Administering a bolus of 500 mL normal saline b. Administering a bolus of 250 mL of 5% albumin c. Slowing the inlet/return rate d. Administering diphenhydramine

42. The first line of transfusion/infusion therapy in the rapidly bleeding patient should

be:

a. whole blood transfusion b. packed red blood cell transfusion c. transfusion of packed red blood cells and fresh frozen plasma d. infusion of crystalloid solution

43. A1 and A2 red blood cells differ in the which of the following properties?

a. quantity of A antigen b. agglutination by Dolichos biflorus lectin c. presence of unique A antigens on A1 cells d. all of the above

44. The first step in dealing with a suspected hemolytic transfusion reaction should

be: a. send a properly labelled specimen to the blood bank b. evaluate the patient c. stop the transfusion d. administer crystalloid

Page 33: CP RIse Review

45. Which statements about the weak D phenotype is true?

a. Weak D red cells may be safely transfused to Rh negative patients. b. Patients with the weak D phenotype must receive weak D donor cells if

they have anti-D. c. The weak D phenotype may be produced by the absence of one or more

epitopes of the D antigen. d. It is difficult to find compatible blood for patients with antibodies specific

for a D epitope.

46. On the red blood cell, the AB antigens are found predominantly as part of which structures?

a. glycolipids b. actin * spectrin * 4.1 complex c. glycoproteins d. glycophorin A

47. Mothers of infants with ABO-HDN are more likely to be of which blood group?

a. A b. B c. AB d. O e. Equally likely among ABO groups

48. A 33-year-old Rh(D) negative woman (G2P1) is seen for a routine antenatal visit

at 17 weeks into her second pregnancy and is found to have an anti-D titer of 16. The most appropriate recommendation is to:

a. Repeat titer in 1 month b. Follow titers every 2 weeks until > 64 c. Administer Rh immune globulin. d. Perform amniocentesis. e. Perform intrauterine transfusion.

49. A 68-year-old, O Rh(D) negative man is found to have anti-D in his serum on his

pre-operative work-up for repair of an abdominal aortic aneurysm. Post-operatively, he is receives 6 units of platelet concentrates, 4 of which are Rh(D) positive and 2 of which Rh(D) negative. Your recommendation to the surgeon should be:

a. Administer Rh immune globulin. b. Repeat the platelet count q 4 h to be sure the incompatible platelets are not

rapidly cleared.

Page 34: CP RIse Review

c. No intervention is required. d. Perform RBC exchange transfusion with Rh-negative red cells. e. Perform therapeutic plasma exchange to remove the anti-D.

50. Match the isoagglutinins with the red cell phenotype.

O a. anti-A A b. anti-A, anti-B, anti-A,B B c. anti-A1 AB d. anti-B e. anti-O f. none

51. Pathologic cold autoantibodies usually have specificity for which of the following

antigens?

a. P and I/i b. Le and I/i c. Se and I/i d. P and Le

52. Which of the following is the safest transfusion of packed red blood cells?

Assume that the recipients do not have any unexpected red cell alloantibodies. Donor Recipient a. O Rh+ O Rh- b. A Rh- AB Rh+ c. A Rh- O Rh- d. AB Rh- O Rh+

53. The Rh null phenotype is characterized by:

a. short red cell half life b. presence of potent anti-D antibodies c. absence of all Rh antigens on red cells but relative abundance on

leukocytes d. premature loss of homozygous fetuses

54. Which statement about warm red cell autoantibodies is false?

a. they typically cause extravascular hemolysis b. it is not necessary to determine their specificity before transfusion of a

patient c. they are usually polyclonal d. they always cause hemolysis when associated with a lymphoid neoplasm

Page 35: CP RIse Review

55. Which transfusion complication is not attributed to leukocytes:

a. febrile, non-hemolytic transfuson reactions b. transfusion associated graft vs. host disease c. anaphylaxis d. Transfusion Related Acute Lung Injury

56. Which factor(s) contributing to hemolytic disease of the newborn are removed during exchange transfusion?

a. Antigen. b. Antibody c. Antigen and antibody. d. Antigen and bilirubin. e. Antigen, antibody, and bilirubin.

57. Intravascular hemolytic transfusion reactions are most commonly due to

antibodies directed against antigens in which of the following blood group systems?

a. ABO. b. Duffy. c. Kell. d. Kidd e. Rh.

58. A 4-year-old boy with ALL has received chemotherapy and has a hematocrit of

20%. He is transfused with a 150-mL aliquot of RBCs without complications. The following day, 20 minutes after the transfusion of a second 150 mL aliquot of RBCs is initiated, he began to cough and complain of difficulty breathing. The nurse discovered that his temperature had gone up by 2C but his other vital signs remained stable. His respiratory status continued to decline and he was intubated. The most likely reason for his apparent reaction to transfusion is:

a. His underlying clinical condition. b. Volume overload. c. Transfusion-related acute lung injury (TRALI). d. An anaphylactic transfusion reaction. e. A hemolytic transfusion reaction.

59. The most common cause of fever occurring during transfusion is:

a. Intravascular hemolytic transfusion reaction. b. Transfusion-related acute lung injury (TRALI). c. Febrile nonhemolytic transfusion reaction d. Posttransfusion purpura. e. Patient’s underlying condition.

Page 36: CP RIse Review

60. The aPTT is normal in patients with a deficiency of which of the following

clotting factors?

a. Factor VIII:C b. Factor V c. Factor XIII d. Factor IX:C e. von Willebrand Factor

61. A transfusion recipient most likely to be exposed to which of the following

organisms?

a. HIV b. HCV c. Yersinia enterocolitica. d. Coagulase negative Staphylococci. e. HBV

62. Which of the hepatitis viruses is a DNA viruses?

a. HAV. b. HBV. c. HCV. d. HDV. e. HEV.

63. Which of the following components can transmit CMV the most effectively?

a. FFP. b. Cryoprecipitated AHF. c. Platelets Pheresis, Leukocytes Reduced. d. RBCs. e. Frozen deglycerolized red cells.

64. Which of the following outcomes is associated with minor ABO-mismatched

allogeneic HPC transplants?

a. Decreased risk of GVHD. b. Hemolysis 40 to 60 days after transplantation. c. Hemolysis 7 to 10 days after transplantation. d. Delayed red cell engraftment. e. Delayed granulocyte engraftment.

Page 37: CP RIse Review

65. Which statement comparing unrelated umbilical cord blood vs. unrelated marrow for allogeneic HPC transplantation is not true?

a. No risk to the donor. b. Lower risk of GVHD. c. More rapidly available. d. Greater representation of minorities. e. More rapid platelet engraftment.

66. T-cell depletion of allogeneic HPCs is associated with all of the following except:

a. Decreased incidence of GVHD. b. Decreased incidence of graft-vs-leukemia effect. c. Decreased incidence of early immune reconstitution. d. Decreased incidence of infections. e. Increased incidence of graft failure.

67. A 24-year-old woman, type B Rh(D) positive, undergoes a peripheral blood stem

cell transplant. The donor is type O Rh(D) negative. This is an example of a:

a. Major ABO mismatch. b. Minor ABO mismatch. c. Major-minor ABO mismatch. d. None of the above

68. The test which is least important for matching a kidney donor with a recipient is:

a. ABO typing. b. Rh typing. c. HLA typing. d. Mixed lymphocyte culture (MLC) testing. e. Lymphocytotoxicity testing.

Page 38: CP RIse Review

69. A family has been typed for HLA because one of the children needs an HPC

transplant. The typing results are as follows:

Father: A1, A3 ; B8, B35 Mother: A2, A23; B12,B18 Child #1: A1, A2 ; B8, B12 Child #2: A1, A23; B8, B18 Child #3: A3, A23; B18,BX

Antigen X in child #3 must be:

a. B1 b. B8 c. B12 d. B18 e. B35

70. The antigen system in humans which is the most highly polymorphic is:

a. ABO. b. MNSs. c. Rh. d. HLA. e. HTLA.

71. In the centrifugation chamber in an apheresis instrument, which element of the

blood is found the farthest from the axis of rotation?

a. Plasma. b. Platelets. c. Lymphocytes. d. Red cells. e. Granulocytes.

72. Fresh frozen plasma is indicated for the replacement fluid for which of the

following diseases?

a. Acute inflammatory demyelinating polyneuropathy. b. Myasthenia gravis. c. Thrombotic thrombocytopenic purpura. d. Refsum’s syndrome. e. Catastrophic Antiphospholipid Antibody Syndrome.

Page 39: CP RIse Review

73. The pH of a unit of platelets at the end of the storage period must be at least:

a. 8.2 b. 7.2 c. 6.2 d. 5.2 e. 4.2

74. What percentage of an intravascular plasma component is removed by a 1 blood

volume plasma exchange procedure, assuming no re-equilibration from the extravascular compartment?

a. 90% b. 70% c. 50% d. 30% e. 25%

75. The use of CMV ‘reduced’risk” cellular blood components is recommended for

all of the following patients except:

a. seronegative patients infected with HIV b. seronegative, full-term infants c. seronegative, low birth weight infants d. seroneagtive pregnant women e. intrauterine transfusions

Page 40: CP RIse Review

Answer Key to Review Questions: Blood Banking and Transfusion Medicine

Question Answer Question Answer

1 A 39 D 2 B 40 B 3 D 41 C 4 D 42 D 5 D 43 D 6 E 44 C 7 B 45 C 8 E 46 C 9 D 47 D 10 A 48 D 11 E 49 C 12 A 50 ObAdBaABf 13 B 51 A 14 D 52 Bb 15 E 53 A 16 C 54 D 17 B 55 C 18 A 56 E 19 A 57 A 20 D 58 C 21 E 59 E 22 B 60 C 23 D 61 D 24 B 62 B 25 B 63 D 26 D 64 C 27 B 65 E 28 B 66 D 29 C 67 B 30 E 68 B 31 E 69 E 32 E 70 D 33 A 71 D 34 D 72 C 35 B 73 E 36 C 74 B 37 C 75 B 38 B

Page 41: CP RIse Review

Recommended References:

Blood Banking and Transfusion Medicine

Brecher, Mark E., ed. Technical Manual. 15th ed. Bethesda, MD: American Association of Blood Banks, 2005. A good, readable, basic text. This text and Standards listed below can be ordered from the AABB on-line (www.aabb.org). Standards for blood banks and transfusion services. 23nd ed. Bethesda, MD, American Association of Blood Banks, 2004. A basic reference. Simon TL, Dzik WH, Snyder E, Stowell CP, Strauss RG, eds. Principles of Transfusion Medicine. 3nd edition. Philadelphia, PA: Lippincott, Williams and Wilkins, 2002. A more in-depth text with a stronger focus on medical, as opposed to technical, aspects of transfusion medicine (www.lww.com).

Page 42: CP RIse Review
Page 43: CP RIse Review

Questions for Molecular Diagnostics and Genetics Talk

1. Which of the following syndromes carries the greatest risk for uterine (endometrial) cancer?

a. Cystic fibrosis b. Li-Fraumeni syndrome c. Hereditary breast/ovarian cancer syndrome d. Familial adenomatous polyposis e. Hereditary non-polyposis colorectal cancer

2. Which of the following syndromes is most closely associated with elevated levels

of hemoglobin A2?

a. Canavan disease b. Tay-Sachs disease c. Alpha-thalassemia d. Beta-thalassemia e. Sickle cell anemia

3. The following pedigree best exemplifies which type of inheritance?

II-4

III-5 III-6

II-3

I-3 I-4

II-5

III-8 III-9

II-6

III-10 III-11III-7

II-2II-1

I-1 I-2

III-1 III-2 III-3 III-4

a. Autosomal dominant inheritance b. Autosomal recessive inheritance c. X-linked recessive inheritance d. Mitochondrial inheritance e. Multifactorial inheritance

Page 44: CP RIse Review

4. From the pedigree in question 3, the normal allele is represented by A and the

mutated allele is a. Which of the following reflects the genotype of individual II-3?

a. aa b. Aa c. AA d. (a) or (b) e. (b) or (c)

5. From the pedigree in question 3, what is the genotype of individual III-5?

a. aa b. Aa c. AA d. (a) or (b) e. (b) or (c)

6. A male is affected with a mild autosomal dominant trait. What is the risk that his

first daughter will be affected with the same trait?

a. 100% b. 75% c. 50% d. 25% e. 0%

7. From question 6, this same male has three affected daughters and three affected sons. What is the risk that his fourth son will be affected with the same trait?

a. 100% b. 75% c. 50% d. 25% e. 0%

8. Chromosomes in karyotypes are typically fixed at which of the following mitotic

stages?

a. Interphase b. Prophase c. Metaphase d. Anaphase e. Telophase

Page 45: CP RIse Review

9. Which of the following karyotypes is an example of a human aneuploidy?

a. 46, XY b. 69, XXX c. 23, X d. 68, XY e. 69, XXY

10. A pregnant woman comes to your office and reports the following family history.

Based on this pedigree, what is the most likely pattern of inheritance of the condition in her family? (The arrow points to the patient. The diamond represents the patient’s fetus, sex unknown.)

P

a. Autosomal dominant b. Autosomal recessive c. X-linked recessive d. Mitochondrial e. Multifactorial

11. Based on the above pedigree, what is the risk that the proband’s fetus is affected?

a. 100% b. 67% c. 50% d. 25% e. 0%

Page 46: CP RIse Review

12. Based on the pedigree from question 10, amniocentesis is performed to determine the sex of the proband’s fetus. The karyotype is reported as 46, XX. What is the risk that her fetus will be affected?

a. 100% b. 67% c. 50% d. 25% e. 0%

13. Based on the pedigree from question 10, amniocentesis is performed to determine

the sex of the proband’s fetus. The karyotype is reported as 46, XY. What is the risk that her fetus will be affected?

a. 100% b. 67% c. 50% d. 25% e. 0%

14. Which of the following techniques is most frequently used in the amplification of

RNA?

a. FISH b. LCR c. PCR d. TMA e. RT-PCR

15. DNA fingerprinting identifies individuals using polymorphisms in which of the

following repetitive DNA sequences?

a. VNTRs b. L1 family c. Satellite DNA d. Alu family e. Trinucleotide repeats

Page 47: CP RIse Review

Answers: for Molecular Diagnostics and Genetics Talk – John Branda, MD 1. (E) Hereditary non-polyposis colorectal cancer 2. (D) Beta-thalassemia 3. (B) Autosomal recessive inheritance 4. (B) Aa 5. (A) aa 6. (C) 50% 7. (C) 50% 8. (C) Metaphase 9. (D) 68, XY 10. (C) X-linked recessive 11. (D) 25% 12. (E) 0% 13. (C) 50% 14. (E) RT-PCR 15. (A) VNTRs

Page 48: CP RIse Review
Page 49: CP RIse Review

Hematology I & II Questions – Daniel Mais, MD, FASCP

1. Which is least likely to express FMC-7?

A. Follicular lymphoma

B. Mantle cell lymphoma

C. Small lymphocytic lymphoma

D. Prolymphocytic leukemia

E. Hairy cell leukemia

2. Follicular lymphoma usually is negative for:

A. CD10

B. BCL-6

C. BCL-2

D. CD43

E. CD22

3. All of the following are typical for hairy cell leukemia, except:

A. Cytopenias and splenic involvement

B. Ribosome-lamellar complexes and TRAP positivity

C. Prominent nucleoli and lymph node involvement

D. Blood lakes and mast cells

E. Expression of CD103 and CD11c

4. All of the following are true of Burkitt’s lymphoma, except:

A. Surface immunoglobulin (sIg) and CD10 are usually expressed

B. Endemic (African) Burkitt’s is strongly associated with EBV

C. Ki67 (PCNA) and BCL-6 are usually expressed

D. Prominent nucleoli and cytoplasmic vacuoles are commonly seen

E. CD34 and Tdt are usually expressed

Page 50: CP RIse Review

5. Hemophagocytic syndrome is typically associated with which infectious agent?

A. Hantavirus

B. Syphilis

C. Tuberculosis

D. Plasmodium falciparum

E. Epstein-Barr virus (EBV)

6. Which of the following often displays acanthocytosis?

A. Absence of Kx antigens (Mcleod phenotype)

B. Uremia

C. Gastric carcinoma

D. Pyruvate kinase deficiency

E. Glucose-6-phosphate dehydrogenase deficiency

7. In a patient with erythrocytosis, features consistent with Polycythemia Vera include

all of the following, except:

A. Increased RBC mass

B. Decreased PaO2

C. Increased LAP score

D. Decreased EPO

E. Decreased serum iron

8. A patient with an artificial heart valve presents with anemia, elevated LDH, and

elevated unconjugated bilirubin. On a peripheral blood smear which would you expect to

find?

A. Microspherocytes

B. Sickle cells

C. Schistocytes

D. Macro-ovalocytes

E. Rouleaux

Page 51: CP RIse Review

9. Which is false?

A. The serum EPO should be elevated in chronic lung disease.

B. The serum EPO should be elevated in hemolytic anemia.

C. The serum EPO should be elevated in polycythemia vera.

D. The serum EPO should be elevated in cerebellar hemagioblastoma.

E. The serum EPO should be elevated in high oxygen-affinity hemolgobins.

10. All of the following are consistent with a chronic hemolytic disorder, except:

A. Increased LDH

B. Decreased bilirubin

C. Decreased haptoglobin

D. Increased reticulocytes

E. Gallstones

11. All of the following are true, except:

A. Hereditary spherocytosis may be due to defects in spectrin, ankyrin, or band 3.

B. Hereditary spherocytosis is most prevalent in sub-Saharan Africa.

C. Hereditary spherocytosis in inherited as an autosomal dominant trait.

D. Hereditary spherocytosis displays a positive osmotic fragility test.

E. Hereditary spherocytosis displays splenomegaly and jaundice.

12. Expected hematologic findings in the Hantavirus pulmonary syndrome include all of

the following, except:

A. Thrombocytopenia

B. Neutrophilia

C. Increased hematocrit

D. Increased lymphoblasts

E. Toxic granulation

Page 52: CP RIse Review

13. The immunophenotype that characterizes acute promyelocytic leukemia (APML)

includes:

A. CD34 positive

B. CD13 negative

C. Tdt positive

D. HLA-DR negative

E. CD235 positive

14. Hematogones in the marrow display:

A. Uniform expression of CD19

B. A tendency to cluster

C. Uniform expression of CD20

D. Increased numbers in children

E. Uniform expression of CD34

15. Which of the following typically involves the marrow in the form of paratrabecular

lymphoid aggregates?

A. Burkitt’s leukemia

B. Chronic lymphocytic leukemia

C. Mantle cell lymphoma

D. Follicular lymphoma

E. Splenic lymphoma with villous lymphocytes

16. In T cell neoplasms, which of the following is true?

A. NK-cell neoplasms usually express CD3

B. Adult T cell lymphomas (ATCL) usually express CD25

C. CD8 expression is typical of Sezary syndrome

D. Alk expression in ALCL implies a worse prognosis

E. HTLV-1 is the etiologic agent in peripheral T cell lymphoma (PTCL)

Page 53: CP RIse Review

17. All of the following are correct associations, except

A. Mantle cell lymphoma and t(11;14)

B. Lymphoplasmacytic lymphoma and t(9;14)

C. Follicular lymphoma and t(14;18)

D. Anaplastic large cell lymphoma and t(2;5)

E. Chronic lymphocytic leukemia and trisomy 13

18. All of the following are associated with a milder clinical course than typical sickle

cell (SS) disease, except:

A. S-α thalassemia

B. S-β thalassemia

C. S-C (SC disease)

D. S-HFPH (S-hereditary persistence of fetal hemoglobin

E. S-A (sickle cell trait)

19. All of the following are associated with Heinz bodies, except:

A. Glucose-6-phosphate dehydrogenase deficiency

B. Hemoglobin Hasharon

C. Sulfhemoglobinemia

D. Hemoglobin H disease

E. Pyruvate kinase deficiency

20. Paratrabecular lymphoid aggregates are most likely to represent

A. Chronic lymphocytic leukemia

B. Follicular lymphoma

C. Mantle cell lymphoma

D. Hairly cell leukemia

E. Burkitt’s leukemia

Page 54: CP RIse Review

21. This gate is appropriate for:

A. Mature lymphocytes

B. Plasma cells

C. Monocytes

D. Blasts

E. Neutrophils

22. A leukemia composed of these cells is:

A. Surface immunoglobulin positive

B. Tdt positive

C. CD34 positive

D. t(4;11) positive

E. CD19 negative

23. When this is seen on flow cytometry, all of the following are true, except:

A. The differential diagnosis includes chronic lymphocytic leukemia

B. The differential diagnosis includes mantle cell lymphoma

C. The differential diagnosis includes marginal zone lymphoma

D. CD23 expression will be important in this differential

E. FMC-7 expression will be important in this differential

24. In a patient with CLL, these cells:

A. Tend to express strong CD5

B. Tend to express strong surface immunoglobulin.

C. Are often present but should be less than 11%.

D. Regardless of count, do not impact prognosis.

E. May express TRAP

Page 55: CP RIse Review

25. This gate is appropriate for:

A. Mature lymphocytes

B. Plasma cells

C. Monocytes

D. Blasts

E. Neutrophils

Page 56: CP RIse Review

ANSWERS: Hematology I & II – Daniel Mais, MD, FASCP

1. C

2. D

3. C

4. E

5. E

6. A

7. B

8. C

9. C

10. B

11. B

12. E

13. D

14. A

15. D

16. B

17. E

18. B

19. E

20. B

21. D

22. A

23. C

24. C

25. B

Page 57: CP RIse Review

ASCP Immunology Review Course Study Questions 1-100 1) The most common peripheral blood lymphocyte is the: A) CD19 positive B cell B) CD8 positive T cell C) CD4 positive T cell D) CD56 positive NK cell 2) All of the following are associated with Goodpasture's syndrome EXCEPT: A) Antibody directed against alpha-3 chain of type IV collagen B) Mesangial IgA deposition C) Linear deposits of IgG along the glomerular basement membrane D) Pulmonary hemorrhage and hematuria 3) Primary biliary cirrhosis is associated with all of the following EXCEPT: A) Anti-mitochondrial antibodies B) Increased IgM levels C) Anti-smooth muscle antibodies D) Female predominance 4) Pemphigus vulgaris is characterized by all of the following EXCEPT: A) A common presentation is a history of recurrent oral sores B) Autoantibodies to hemidesmosomal proteins are present C) Associated with celiac disease, may respond to a gluten free diet D) Nikolsky's sign 5) All of the following are examples of type III hypersensitivity reactions EXCEPT: A) SLE B) Serum sickness C) Arthus reactions D) Contact dermatitis 6) Transient hypogammaglobulinemia of infancy is characterized by all the following EXCEPT: A) Reduced levels of serum IgG and IgA B) Normal numbers of CD19 positive B cells in blood C) Typically presents in the first two months of life D) Have a history of recurrent upper respiratory tract infections

Page 1 of 18

Page 58: CP RIse Review

7) Hyperimmunoglobulin M syndrome is characterized by which ONE of the following: A) Female predominance B) Autosomal recessive inheritance C) Markedly decreased levels of CD19 positive B cells D) Deficiency in CD40 ligand (CD154) on T cells 8) DiGeorge's syndrome is associated with all of the following EXCEPT: A) Hypocalcemia B) Lack of a thymus C) Autosomal dominant inheritance D) Failure of development of 3rd and 4th pharyngeal pouches 9) The following lab results would be expected in hereditary angioedema EXCEPT: A) Decreased C4 B) Decreased Factor B C) Decreased functional C1 esterase inhibitor D) Decreased CH50 10) The most common inherited complement deficiency is deficiency of: A) C1 B) C2 C) C4 D) C9 11) All of the following are features of MGUS (monoclonal gammopathy of unknown significance) EXCEPT: A) No Bence Jones protein in urine B) No lytic bone lesions C) >50% will develop multiple myeloma within 10 years D) No hypercalcemia 12) Type II cryoglobulins consist of: A) Monoclonal IgG B) Polyclonal IgG and polyclonal IgM C) Monoclonal IgM and polyclonal IgG D) Monoclonal IgA 13) Type II cryoglobulinemia is associated with all of the following EXCEPT: A) Patients often infected with HCV

Page 2 of 18

Page 59: CP RIse Review

B) Often have high-level rheumatoid factor activity C) Also known as "mixed cryoglobulinemia" D) Strong association with Waldenström's macroglobulinemia 14) All of the following are associated with Sjogren's syndrome EXCEPT: A) Antibodies verus SS-A (Ro) and SS-B (La) are commonly seen B) Positive ANA in 100% C) Female predominance D) Dry mouth and dry eyes 15) Which of the following sets of lab results are most associated with Mixed Connective Tissue Disease? A) Homogeneous ANA at 1:5120, anti-Smith positive B) Centromere pattern ANA at 1:1280 C) Speckled ANA at 1:1280, High titer anti-RNP antibodies D) Speckled ANA at 1:1280, High titer Ro and LA antibodies 16) Given the following set of lab results which diagnosis is most likely? AST 450 IU/dL (ref range 10-40 U/L) ALT 650 IU/dL (ref range 10-55 U/L) Alkaline phosphatase 140 IU/dL (ref range 40-115 U/L) IgG 2,420 mg/dL (ref range 600-1400 mg/dL) Anti-smooth muscle 1:1280 (high titer) Anti-mitochondrial antibodies 1:40 (low titer) Anti-LKM antibodies: Negative Anti-HCV ELISA: Negative A) Primary biliary cirrhosis B) Autoimmune hepatitis type I C) Autoimmune hepatitis type II D) Viral hepatitis 17) Type II autoimmune hepatitis is characterized by which of the following antibodies: A) Anti-mitochondrial antibodies B) Anti-smooth muscle antibody C) Anti-LKM-1 (Liver-kidney-microsomal) antibodies D) Anti- Ku 18) Patient is a 60-year-old male with a history of recurrent mouth sores and flaccid skin blisters. A biopsy of peri-lesional mucosa will likely show:

Page 3 of 18

Page 60: CP RIse Review

A) IgG and C3 in a “honeycomb pattern” B) IgA deposition at the dermal papillae C) No immune deposits D) IgG and C3 in a linear band at the dermal-epidermal junction 19) Type II cryoglobulinemia is most commonly due to: A) SLE B) IgG myeloma C) Autoimmune hepatitis D) HCV 20) Light chain deposition disease (LCDD) can be differentiated from amyloidosis by: A) Lack of renal failure in LCDD B) Lack of Congo Red staining in LCDD lesions C) The type of monoclonal light chain (kappa vs. lambda) D) Presence of nephrotic syndrome in amyloidosis 21) In immunoglobulin heavy chain diseases the light chains are not found in association with the heavy chains since: A) The light chains have failed to rearrange productively in the developing B cells B) The light chains are deleted C) The light chains are mutated D) Internal deletions are present in the monoclonal heavy chains 22) Regarding Bence Jones proteins (BJP) which of the following is true: A) Found in 2% of multiple myeloma patients B) A negative urine dipstick for protein rules out Bence Jones proteins in the urine C) BJP are polyclonal light chains that aggregate in the urine D) BJP may be associated with AL amyloidosis 23) Which paraprotein is most commonly elevated in hyperviscosity syndrome? A) IgA B) IgG C) IgM D) IgD 24) The protein involved in the most common form of secondary amyloidosis is: A) Prealbumin B) Lambda light chain C) Kappa light chain

Page 4 of 18

Page 61: CP RIse Review

D) Serum amyloid A 25) The diagnostic criteria for Allergic Bronchopulmonary Aspergillosis include all of the following EXCEPT: A) History of asthma B) Elevated IgE C) Serum precipitating antibodies to Aspergillis D) Skin test reactivity to Aspergillis 26) The following statements regarding Hypersensitivity Pneumonitis (HP) are true EXCEPT: A) Negative serum precipitins rule out the diagnosis of HP B) Farmer’s lung is an example of HP C) Pigeon fancier’s disease is an example of HP D) Serum precipitins are IgG antibodies specific for HP antigens 27) Contraindications to skin prick testing include all of the following EXCEPT: A) Potentially dangerous allergens (e.g. bee venom) B) Children less than 10 years old C) Patients on anti-histamines D) Patients with dermatographism 28) The type of anti-neutrophil cytoplasmic antibody (ANCA) that may be found in association with ulcerative colitis is which of the following: A) cANCA B) pANCA C) Atypical pANCA D) None of the above 29) Which of these diseases is not commonly associated with anti-neutrophil cytoplasmic antibody (ANCA) positivity? A) Wegener’s Granulomatosis B) Polyarteritis Nodosa C) Microscopic Polyangiitis D) Churg-Strauss 30) When tested for the ANCA in the indirect immunofluorescence assay, a patient’s serum stains the cytoplasm of ethanol-fixed human neutrophils. What is the most likely antigen? A) Cathepsin G

Page 5 of 18

Page 62: CP RIse Review

B) Proteinase 3 C) Myeloperoxidase D) Elastase 31) Rheumatoid factor is commonly elevated in all of the following diseases EXCEPT: A) Sjogren’s syndrome B) Rheumatoid arthritis C) SLE D) Celiac disease 32) Which of the following assays is most specific for rheumatoid arthritis? A) Rheumatoid factor B) Anti-Citrullated Cyclic Peptide C) C-reactive protein D) Anti-Scl-70 33) A patient presents with proximal muscle weakness, a scaly rash on the extensor surfaces of her fingers, and the following labs: ANA positive at 1:1280 on HEp-2 cells, speckled pattern Anti-Jo-1 positive Muscle biopsy shows perivascular, perifascicular infiltrates What is the most likely diagnosis? A) Polymyositis B) Dermatomyositis C) Inclusion body myositis D) SLE-related myositis 34) The most serious complication of Sjogren’s syndrome is: A) Lymphoma B) Renal failure C) Pulmonary hypertension D) Dry eyes 35) Autoantibodies commonly seen in scleroderma include all of the following EXCEPT: A) Anti-Scl-70 B) Anti-centromere C) Anti-RNA polymerase I D) Anti-Smith 36) When tested for the ANCA in the indirect immunofluorescence assay, a patient’s serum stains ethanol-fixed human neutrophils in a perinuclear distribution. When the

Page 6 of 18

Page 63: CP RIse Review

serum is tested on formalin-fixed neutrophils, staining is seen in a cytoplasmic distribution. The patient’s antibody is the most likely directed against which antigen? A) Myeloperoxidase B) Elastase C) Proteinase 3 D) None of the above 37) What is the antibody most highly associated with neonatal Lupus? A) Anti-dsDNA B) Anti-Smith C) Anti-Ro D) Anti-Scl-70 38) Infections with all of the following organisms are associated with T cell deficiency EXCEPT: A) Neisseria B) Candida C) Listeria monocytogenes D) Pneumocystis 39) What syndrome is associated with defects in the Fas-dependant apoptotic cell death pathway? A) Autoimmune lymphoproliferative syndrome (ALPS) B) X-linked Lymphoproliferative Syndrome C) Severe Combined Immunodeficiency D) Job’s Syndrome 40) All of the following are true about X-linked agammaglobulinemia EXCEPT: A) This disease is also known as Bruton’s agammaglobulinemia B) IgA and IgM are absent C) The disease is caused by mutations in the B cell tyrosine kinase (Btk) D) Patients have normal numbers of CD19 positive B cells in the peripheral blood 41) In acute phase reactions the following are typically observed on serum protein electrophoresis EXCEPT: A) An increased transferrin band B) An increased alpha-1-anti-trypsin band C) An increased C3 band D) Increased alpha-2 zone proteins

Page 7 of 18

Page 64: CP RIse Review

42) The allelic phenotype in alpha-1-antitrypsin deficiency that is at highest risk for emphysema is: A) PiMM B) PiSS C) PiSZ D) PiZZ 43) A patient with Waldenstrom’s has a serum viscosity of 3.5 (nl 1.4-1.8). What symptoms would this patient most likely have that are attributable to the increased viscosity? A) Stroke B) Diplopia C) Epistaxis D) No symptoms 44) Type III cryoglobulin precipitates are composed of: A) Polyclonal IgM and polyclonal IgG B) Monoclonal IgM and polyclonal IgG C) Monoclonal IgG, IgA, or IgM D) Polyclonal IgM and monoclonal IgG 45) All of the following are true of bullous pemphigoid EXCEPT: A) Most patients typically are in their 2nd and 3rd decades B) Presents with pruritic, tense bullae on skin C) Caused by IgG antibodies to hemidesmosomal antigens D) Immunofluorescence demonstrates IgG and C3 deposition in a linear band and the dermal-epidermal junction 46) The following are true regarding myeloperoxidase deficiency EXCEPT: A) Patients are unable to form hypochlorous acid in neutrophil granules B) Inheritance is autosomal recessive C) The diagnostic procedure of choice is staining of PMNs for myeloperoxidase D) Most patients die of overwhelming Candida infections 47) A newborn presents with a history of delayed separation of the umbilical cord, leukocytosis and numerous abscesses. What diagnosis is suggested by these findings? A) Chronic mucocutaneous candidiasis B) Leukocyte adhesion deficiency type I C) Leukocyte adhesion deficiency type II

Page 8 of 18

Page 65: CP RIse Review

D) Chronic granulomatous disease 48) The screening test for chronic granulomatous disease is: A) Flow cytometry for CD18 B) Flow cytometry for CD59 C) Nitroblue tetrazolium test D) Staining of PMNs for myeloperoxidase 49) These autoantibodies are most highly associated with Mixed Connective Tissue disease: A) Anti-U1-RNP B) Anti-U3-RNP (fibrillarin) C) Anti-RNA polymerase I D) Anti-Ro 50) The autoantibody that may be used to follow the activity of nephritis in SLE patients is: A) Anti-Smith B) Anti-dsDNA C) Anti-histone D) Anti-Ro 51) The test with the highest negative predictive value for SLE: A) Anti-Smith B) Anti-dsDNA C) ANA by indirect immunofluorescence on HEp-2 cells D) Anti-Ro 52) Job's syndrome is characterized by all of the following EXCEPT: A) High levels of IgE and eosinophilia B) Recurrent staphylococcal abscesses C) X-linked recessive inheritance D) Eczema 53) Bare lymphocyte syndrome is characterized by all of the following EXCEPT: A) Caused by mutations within the MHC class II genes (DR, DP, DQ) B) Presents in first year of life with recurrent diarrhea and infections C) Lack of CD4+ T cells D) Lack of MHC class II on B cells

Page 9 of 18

Page 66: CP RIse Review

54) All of the following are characteristic of common variable immunodeficiency EXCEPT: A) Reduced numbers of CD19+ B cells in peripheral blood B) Variable age of presentation, infancy to 4th decade C) Recurrent respiratory infections with encapsulated bacteria D) Low levels of circulating IgG and IgA 55) All of the following are true of IgG subclasses EXCEPT: A) IgG3 is involved in immune responses to protein antigens B) IgG2 is involved in immune responses to polysaccharide antigens C) Patients with Ig subclass deficiency have decreased levels of total Ig D) Diagnosis of Ig subclass deficiency requires both decreased subclass and evidence of functional antibody impairment 56) The majority of patients with plasmacytomas have all of the following EXCEPT: A) Normal calcium levels B) Sheets of plasma cells on biopsy of the affected site C) Normal hematocrit levels D) High levels of M-component in the serum 57) Plasma cell leukemia can be differentiated from multiple myeloma (MM) by which ONE of the following: A) Better prognosis than MM B) In plasma cell leukemia >20% of peripheral blood mononuclear cells are monoclonal plasma cells C) Higher level of M component than in MM D) Presence of a characteristic genetic abnormality in plasma cell leukemia that is not found in MM 58) The following diseases are characterized by decreased C3 levels EXCEPT: A) Lupus nephritis B) Goodpasture's syndrome C) Post-streptococcal glomerulonephritis D) Cryoglobulinemia 59) The following diseases are characterized by decreased C4 levels EXCEPT: A) Gram negative sepsis B) Hereditary angioedema C) Serum sickness D) Lupus nephritis

Page 10 of 18

Page 67: CP RIse Review

60) Total hemolytic complement levels are decreased in all of the following EXCEPT: A) Hereditary angioedema B) Type I glomerulonephritis C) Acute phase reactions D) Paroxysmal nocturnal hemoglobinuria 61) A complement fixation test is performed in order to detect histoplasmosis antibodies. The tube to which the patient's serum was added is hemolyzed. Is this a positive or negative test? A) Positive B) Negative 62) Nephelometry works on the principle of: A) Light absorption by immune complexes B) Light scatter by immune complexes C) Blockage of light transmittance by immune complexes D) Light emitted by immune complexes 63) In serum protein electrophoresis the pattern seen in cirrhosis is which of the following: A) ↓A1AT, ↓Albumin, ↓A2M, ↑IgG B) ↑A1AT, ↓Albumin, ↑A2M, ↑IgG C) ↑A1AT, ↓Albumin, ↓A2M, ↓IgG D) ↓A1AT, ↓Albumin, ↓A2M, ↓IgG 64) All of the following are true regarding celiac disease EXCEPT: A) Anti-endomysial IgA and anti-tissue transglutaminase IgA antibodies are sensitive tests for celiac disease B) Serologic testing plays no role in monitoring response to treatment C) There is a higher proportion of selective IgA deficiency in celiac disease than in the general population D) Celiac disease is caused by immune mediated destruction of the absorptive surface of the intestine 65) The radioallergosorbent (RAST) test: A) Tests for allergen specific IgG B) Tests for allergen specific IgE C) Tests for total IgE

Page 11 of 18

Page 68: CP RIse Review

D) Involves scratching the skin with the allergen of interest 66) All of the following are true regarding selective IgA deficiency EXCEPT: A) Selective IgA deficiency is the most common primary immunodeficiency B) The majority of patients present with intractable diarrhea and recurrent infections C) A subset of patients may have accompanying IgG subclass deficiencies D) Patients have normal number of CD19+ B cells 67) All of the following are true regarding ataxia-telangiectasia EXCEPT: A) It is an autosomal recessive disease caused by mutations in the ATM gene B) Ataxia in childhood due to cerebellar degeneration is a common presenting symptom C) Variable immunodeficiency is present D) No increased incidence of malignancy is observed 68) Laboratory testing supportive of a diagnosis of multiple sclerosis include all the following EXCEPT: A) Presence of oligoclonal banding in the CSF B) An increased IgG index in the CSF C) An increased IgG to albumin ratio in the CSF D) Decreased IgG synthesis in the CSF 69) All of the following are true regarding Wiskott-Aldrich syndrome EXCEPT: A) It is an X-linked disease due to mutations in the Wiskott-Aldrich syndrome protein B) Patients have thrombocytosis with small dysfunctional platelets C) IgM and IgG levels are low D) Patients may present with eczema and evidence of immunodeficiency 70) Patient has a history of recurrent attacks of abdominal pain (diagnosed as peritonitis, culture negative) and fever since age 6. During attacks the CRP is noted to be elevated. The patient is now 30 and has nephrotic syndrome secondary to amyloidosis. What is the most likely diagnosis? A) Light chain deposition disease B) Familial Mediterranean fever C) Multiple myeloma D) POEMS syndrome 71) In question #70 the patient’s type of amyloid would be expected to be: A) AL amyloid B) AF amyloid C) AA amyloid

Page 12 of 18

Page 69: CP RIse Review

D) None of the above 72) Antibodies associated with paraneoplastic cerebellar degeneration include: A) Anti-Yo B) Anti-myelin C) Anti-Purkinje cell D) Anti-Hu 73) Reticular dysgenesis can be distinguished from other forms of SCID by: A) The complete absence of T and B cells B) An increase in T cells C) The lack of NK cells D) The lack of granulocytes 74) The patient is a 49 year old female complaining of arthralgias, dry mouth and dry eyes. Testing reveals a strongly positive ANA (>1:5120, speckled). Assays for anti-Smith, anti-dsDNA, anti-Ro, anti-La, anti-U1RNP, and anti-Scl-70 are all negative. Which of the following statements is most likely to be correct? A) The patient has a 95% chance of having a connective tissue disease B) SLE can be ruled out due to the patient’s negative assays for anti-Smith and anti-dsDNA C) Sjogren’s syndrome may be diagnosed in this patient if she meets the diagnostic criteria D) Mixed connective tissue disease may be diagnosed in this patient 75) All of the following are true regarding linear IgA dermatosis EXCEPT: A) Oral and ocular lesions are common B) Linear anti-IgA staining of the basement membrane is found on immunofluorescence of peri-lesional skin C) Biopsy of lesions demonstrates a neutrophilic infiltrate D) Serum IgA levels are low 76) A patient with IDDM who subsequently develops Graves disease and adrenal insufficiency should be suspected of having: A) Polyglandular autoimmune syndrome type I B) Polyglandular autoimmune syndrome type II C) Multiple endocrine neoplasia type II D) None of the above

Page 13 of 18

Page 70: CP RIse Review

77) A wavy band is noted on serum protein electrophoresis located between transferrin and C3. The patient is also noted to have a decreased albumin and high-level proteinuria. What is the protein that corresponds to the wavy band? A) C-reactive protein B) IgA M component C) Lipoprotein B D) Lipoprotein A 78) A 55-year-old male patient presents with an acutely painful right knee. Synovial fluid analysis demonstrates an inflammatory joint fluid with a negative gram stain. Crystal analysis shows numerous birefringent needle shaped crystals. Which ONE of these statements is correct: A) The etiology of the disease in this patient is most likely to be overproduction of uric acid. B) The knee is the joint most commonly affected in this disease C) The crystals would be expected to show strong positive birefringence D) The crystals in this patient would be expected to be yellow when parallel to the axis of the red compensator on polarized microscopy 79) In a patient with scleroderma anti-Scl-70 positivity: A) Is present in 80% of patients with scleroderma B) Correlates with an increased risk of lung involvement C) Correlates with an increased risk of scleroderma renal crisis D) Is associated with CREST syndrome 80) All of the following are true regarding alpha heavy chain disease EXCEPT: A) It is the most common type of heavy chain disease B) Lytic lesions are common C) GI tract involvement with MALT like lymphoma may be present D) The bone marrow typically shows no abnormalities 81) The most common form of SCID is due to: A) Mutations in the RAG-1 and RAG-2 genes B) Adenosine deaminase deficiency C) X-linked mutations in the common cytokine receptor D) PNP deficiency 82) The serum protein electrophoresis in nephrotic syndrome is characterized by all of the following EXCEPT:

Page 14 of 18

Page 71: CP RIse Review

A) ↓Albumin B) ↑Alpha-2-macroglobulin C) ↓Transferrin D) ↑Alpha-1-anti-trypsin 83) The percentage of healthy persons with a HEp-2 IIF assay positive at a 1:160 dilution of serum is approximately: A) <1% B) 5% C) 15% D) 33% 84) Hashimoto’s thyroiditis is associated with all of the following EXCEPT: A) High titer anti-thyroid peroxidase antibodies B) High titer thyroid stimulating immunoglobulins C) Elevated TSH D) Lymphocytic infiltration of the thyroid 85) All of the following are associated with Graves disease EXCEPT: A) Anti-TPO and anti-thyroglobulin antibodies may be present B) High titer thyroid stimulating immunoglobulins C) Decreased TSH D) Males and females are equally affected 86) All of the following are true of chronic granulomatous disease (CGD) EXCEPT: A) Patients present before age 5 with infections involving catalase positive organisms B) CGD is due to mutations in the NADPH-oxidase enzyme C) CGD may be associated with deficiency in Kx (McLeod phenotype) D) CGD is X-linked so all CGD patients are males 87) All of the following are true of Chediak-Higashi syndrome EXCEPT: A) Caused by autosomal recessive deficiency of the CHS gene B) Defects in lysosome and vesicle fusion are present in many cell types C) Oculocutaneous albinism is present D) Patient’s RBCs have the “Bombay phenotype” due to lack of H antigen 88) All of the following statements are true of drug-induced ANA positivity EXCEPT:

Page 15 of 18

Page 72: CP RIse Review

A) It is associated with procainamide, hydralazine, and izoniazid B) Antibodies directed against histone H2A-H2B complex are found in most patients C) Anti-Smith antibodies are found in 30% of patients D) Lupus-like symptoms occur in only a small percentage of ANA positive patients 89) All of the following statements are true of Anti-Jo antibodies EXCEPT: A) They are directed against histidyl tRNA transferase B) In the HEp-2 IIF assay they may cause cytoplasmic speckling C) May be associated with a syndrome in polymyositis/dermatomyositis that includes interstitial lung disease D) Anti-Jo-1 is a highly sensitive (>90%) test for polymyositis 90) A patient with the Bombay phenotype (lack of H antigen on their RBCs) also has severe mental retardation and recurrent childhood infections. What is the likely diagnosis? A) Chronic granulomatous disease B) Leukocyte adhesion deficiency type II C) Ataxia-telangiectasia D) Chediak-Higashi syndrome 91) The immunophenotype that is typically found in multiple myeloma is: A) CD19+ CD20+ CD56+ CD138+ B) CD19- CD20- CD56+ CD138+ C) CD19- CD20+ CD56+ CD138- D) CD19+ CD20+ CD56- CD138+ 92) The immunophenotype that is typically found in Waldenstrom’s macroglobulinemia is: A) CD19+ CD20+ CD5- CD10- B) CD19- CD20- CD5+ CD10- C) CD19+ CD20- CD5+ CD10- D) CD19+ CD20+ CD5+ CD10+ 93) Serum viscosity measurements are performed by evaluating a patient’s serum viscosity as compared to the viscosity of: A) Normal human sera B) Water C) Whole blood D) Plasma

Page 16 of 18

Page 73: CP RIse Review

94) A SLE-like disorder may result in patients with genetic deficiencies in all of the following proteins EXCEPT: A) C1 B) C2 C) C4 D) C5 95) Acquired angioedema (AAE) is characterized by all of the following EXCEPT: A) Elevated C1q levels B) Decreased functional and antigenic C1-inhibitor C) In some patients AAE may be caused by an antibody to C1-inhibitor D) May be associated with immune complex diseases or cryoglobulinemia 96) The following diseases may be associated with secondary or reactive amyloidosis EXCEPT: A) Chronic infection B) Multiple myeloma C) Familial Mediterranean fever D) Rheumatoid arthritis 97) A male patient first presented to medical attention at four years of age with a case of severe infectious mononucleosis with hepatic necrosis. Following recovery from this episode, at age 7, the patient is noted to have severe lymphopenia and hypogammaglobulinemia. What is the likely diagnosis? A) Autoimmune lymphoproliferative syndrome (ALPS) B) X-linked lymphoproliferative syndrome (Duncan’s syndrome) C) X-linked SCID D) Omenn’s syndrome 98) The most common monoclonal immunoglobulin found in multiple myeloma is: A) IgAκ B) IgAλ C) IgGκ D) IgGλ 99) Factor B levels are decreased in which ONE of the following: A) Serum sickness B) Gram negative sepsis C) Cryoglobulinemia D) Hereditary angioedema

Page 17 of 18

Page 74: CP RIse Review

100) The albumin band is slurred anodally in a patient’s serum protein electrophoresis. The remainder of the SPEP is within normal limits. This is commonly due to: A) Mutations in the albumin protein B) Binding of bilirubin or medications to albumin C) Defective glycosylation of albumin D) Loading artifact Answer Key: 1. C 2. B 3. C 4. C 5. D 6. C 7. D 8. C 9. B 10. B 11. C 12. C 13. D 14. B 15. C 16. B 17. C 18. A 19. D 20. B

21. D 22. D 23. C 24. D 25. C 26. A 27. B 28. C 29. B 30. B 31. D 32. B 33. B 34. A 35. D 36. A 37. C 38. A 39. A 40. D

41. A 42. D 43. D 44. A 45. A 46. D 47. B 48. C 49. A 50. B 51. C 52. C 53. A 54. A 55. C 56. D 57. B 58. B 59. A 60. C

61. B 62. B 63. A 64. B 65. B 66. B 67. D 68. D 69. B 70. B 71. C 72. B 73. D 74. C 75. D 76. B 77. C 78. D 79. B 80. B

81. C 82. D 83. B 84. B 85. D 86. D 87. D 88. C 89. D 90. B 91. B 92. A 93. B 94. D 95. A 96. B 97. B 98. C 99. B 100. B

Page 18 of 18

Page 75: CP RIse Review

Chemistry Review Questions ⎯ Kent Lewandrowski, M.D. and Anand Dighe M.D., Ph.D.

Page 1 of 13

ASCP Chemistry Review Study Questions 1. All of the following are true of Troponin testing for myocardial infarction except: a) Troponin typically rises after approximately 6 hours following infarction, peaks after 1-1-½

days, and may remain elevated for 7 days or more. b) Troponin is the new “gold standard” marker for acute myocardial infarction. c) Elevations in Troponin may reflect myocardial necrosis, but are not elevated in myocardial

ischemia without necrosis. d) In most applications, Troponin I is favored over Troponin T as the best marker. e) Troponin is measured by immunoassay and the results expressed in ng/mL. 2. All of the following are true of CK/MB except: a) Older electrophoretic assays are obsolete. CK/MB is usually measured by immunoassay. b) The CK/MB relative index is a value used to indicate myocardial as opposed to skeletal

muscle sources for elevation of CK/MB. c) CK/MB is the most abundant isoenzyme in the myocardium. d) Small amounts of CK/MB may be found in skeletal muscle. e) CK contains 3 isoenzymes, none of which are specific for a single tissue source. 3. All of the following are true concerning B-natriuretic peptide except: a) Both BNP and NT-pro BNP are derived from pro BNP. b) BNP is released from cardiac myocytes in response to left ventricular expansion and

pressure overload. c) BNP may be useful in the differential diagnosis of dyspnea, in the diagnosis of congestive

heart failure, and for prognosis in acute coronary syndromes. d) Unlike atrial natriuetic peptide, BNP inhibits naturesis and diuresis. e) BNP levels correlate with the pulmonary artery wedge pressure. 4. All of the following statements are true except: a) Gilbert syndrome is characterized by decreased bilirubin glucuronyl transferase with impaired

bilirubin uptake. b) Gilbert syndrome typically presents with mild asymptomatic unconjugated hyperbilirubinemia. c) Liver biopsy in patients with Gilbert syndrome typically shows a black colored intracellular

pigment. d) Crigler-Najjar syndrome, Type 1, is characterized by abscess of bilirubin glucuronyl

transferrase and is usually fatal in infancy. e) Both Rotor syndrome and Dubin-Johnson syndrome are characterized by conjugated

jaundice. 5. A patient exhibits an elevated alkaline phosphatase, slight elevations in ALT and AST and a

normal bilirubin. The GGT is elevated. The most likely diagnosis in this patient is: a) Paget’s disease of bone. b) Hepatobiliary obstruction. c) Chronic hepatitis. d) Space occupying lesion of the liver. e) Drug induced cholestasis. 6. All of the following are criteria used in the Child-Pugh classification of hepatic status except: a) AST.

Page 76: CP RIse Review

Chemistry Review Questions ⎯ Kent Lewandrowski, M.D. and Anand Dighe M.D., Ph.D.

Page 2 of 13

b) Bilirubin. c) Prothrombin time. d) Albumin. e) Ascites. 7. All of the following are true concerning calcium homeostasis except: a) The N-terminal fragment of PTH contains the active portion of the hormone. The C-terminal

fragment is inactive. b) The formula for corrected calcium is used to adjust the ionized calcium level according to the

pH of the specimen. c) PTH increases calcium resorption and promotes phosphate excretion by the kidney. d) Parathyroid hormone increases the plasma calcium, decreases the plasma phosphate, and

may increase the urinary calcium. e) The role of calcitonin in adult calcium homostasis is unclear, but is probably minor. 8. All of the following are causes of hypercalcemia except: a) Hyperparathyroidism. b) Hypercalcemia of malignancy. c) Thiazide diuretics. d) Milk aklali syndrome. e) Pseudohypoparathyroidism. 9. All of the following are true of magnesium homeostasis except: a) Hypermagnesemia is relatively common. b) Hypomagnesemia may be due to gastrointestinal, renal, and endocrine causes. c) In patients with hypomagnesemia, the urine magnesium value may be used to differentiate

renal from extrarenal etiologies. d) Hypermagnesemia usually occurs in patients with renal impairment with an exogenous

source of magnesium administration. e) Most common laboratory methods for measurement of magnesium are based on colorimetric

complex formation. 10. All of the following are true concerning acid-base homeostasis except: a) Aldosterone promotes sodium resorption and potassium excretion and impairs renal

hydrogen ion excretion. b) Volume depletion may result in contraction alkalosis. c) Titratable acidity cannot be increased in states of acidosis. d) Ammonium excretion can be augmented in patients with acidosis and is the principle

mechanism for accommodating an increased acid load. e) Free acid excretion by the kidney is negligible. 11. All of the following are true concerning acid-base disorders except: a) The most common cause of respiratory alkalosis is hyperventilation due to anxiety. b) Chloride responsive metabolic alkalosis results from salt and water depletion with an HCL

loss. c) Chloride resistant metabolic alkalosis results from mineralocorticoid excess. d) The urine chloride can be used to differentiate chloride responsive from chloride resistant

metabolic alkalosis. e) The urine chloride is typically greater than 20meq per liter in chloride responsive metabolic

alkalosis. 12. All of the following are causes of an increased anion gap metabolic acidosis except:

Page 77: CP RIse Review

Chemistry Review Questions ⎯ Kent Lewandrowski, M.D. and Anand Dighe M.D., Ph.D.

Page 3 of 13

a) Renal failure. b) Ethylene glycol ingestion. c) Lactic acidosis. d) Renal tubular acidosis. e) Paraldehyde administration. 13. All of the following are true of renal tubular acidosis except: a) Type 1 classic RTA occurs when the kidney cannot maximally acidify the urine. b) Type 2 proximal RTA results from bicarbonate wasting. c) Type 4 RTA is due to hyperaldosteronism. d) In type 2 RTA, the fractional excretion of bicarbonate load is typically greater than 15%. e) Patients with renal tubular acidosis typically exhibit a normal anion gap. 14. A patient exhibits a PCO2 of 52 and a PH of 7.35. Using the AHA Rule 1 for interpretation of

acid base disorders, this patient’s condition is classified as: a) Metabolic acidosis with compensation. b) Respiratory acidosis with compensation. c) Uncompensated respiratory acidosis. d) Uncompensated metabolic acidosis. 15. All of the following are true concerning creatinine except: a) Jaffe methods are preferred because enzymatic methods suffer from a number of causes of

interference. b) Causes of an elevated creatinine include increased production, pre-renal azotemia, renal

disease, and post renal obstruction. c) The creatinine clearance can be estimated using the plasma creatinine alone in combination

with the patient’s age, weight, and sex. d) Small amounts of creatinine are secreted by the renal tubular epithelium and as a

consequence the creatinine clearance is usually higher than the true GFR. 16. All of the following are true concerning hyperuricemia except: a) Hyperuricemia due to excessive urate production includes both congenital and acquired

causes. b) Lesch-Nyhan syndrome results from phosphoribosyl synthetase deficiency. c) In patients with hyperuricemia due to excessive urate production, the urine 24 hour urate is

typically greater than 800 mg/ 24 hours. d) In patients with decreased renal urate excretion, the 24 hour urine urate is usually less than

600 mg per 24 hours. e) The routine initial evaluation of hyperuricemia should include assessment of renal function,

serum lipids, estimation of the creatinine clearance, and serum and 24 hour urine urate measurements.

17. All of the following statements concerning acute azotemia are true except: a) Intrinsic renal disease is the most common cause of acute azotemia. b) Acute interstitial nephritis is usually due to drug reactions. c) Hyperkalemia in patients with acute azotemia results from impaired renal potassium excretion

and metabolic acidosis. d) Prerenal azotemia typically exhibits a urine specific gravity greater than 1.016 with a urine

osmolality >500 and a urine <20. e) Hyponatremia in patients with acute azotemia usually results from water retention in excess

of sodium.

Page 78: CP RIse Review

Chemistry Review Questions ⎯ Kent Lewandrowski, M.D. and Anand Dighe M.D., Ph.D.

Page 4 of 13

18. All of the following statements concerning renal function testing and renal disease are true

except: a) Microscopic hematuria is defined as greater than 3-5 red blood cells per high power field. b) Explanations for a false positive dipstick for blood in the urine include hemoglobinuria and

myoglobinuria. c) Up to 20% of patients evaluated for hematuria are ultimately defined as “idiopathic

hematuria”. d) Tubular proteinuria typically consists of albumin and immunoglobulins. e) The finding of red cell casts and dysmorphic red cells in the urine indicates nephritic

syndrome. 19. All of the following statements are true except: a) Common causes of nephritic syndrome include postinfectious glomerulonephritis, drug

associated GN, SLE, bacterial endocarditis, and membranoproliferative GN. b) Fanconi’s syndrome is characterized by a selective tubular defect resulting in isolated

aminoaciduria. c) The most common type of renal stone is composed of calcium oxalate. d) Hyperoxaluria may occur as a rare primary disorder or in association with malabsorption or

inflammatory bowel disease. e) Hypercalcuria may be categorized as resorptive, absorptive, or due to renal hypercalciuria.

Absorptive hypercalciuria can be distinguished from resorptive by virtue of the fact that the urine calcium corrects on a calcium-restricted diet.

20. All of the following statements are true except: a) The extracellular fluid volume virtually always correlates with the effective circulating volume. b) The plasma sodium and osmolality are regulated principally by control of water balance. c) The regulation of volume is controlled principally by regulation of sodium balance. d) Aldosterone acts on the renal collecting tubules to promote sodium and water resorption with

potassium and hydrogen ion excretion. e) Antidiuretic hormone acts on the renal collecting tubules to promote renal water resorption

resulting in a concentrated urine. 21. All of the following are true except: a) Patients with hypernatremia usually exhibit hyperosmolality. b) Hyperosmolality may occur with or without hypernatremia. c) Nephrogenic diabetes insipidous may be familial or acquired and results from an impaired

renal response to ADH. d) A patient with hypernatremia exhibits hypervolemia. The urine osmolality is greater than 700

and the urine sodium is greater than 20. These findings are consistent with mineralocoitocoid excess.

e) Pseudohyponatremia is a laboratory artifact resulting from electrolyte interferences using ion selective electrodes.

22. All of the following are true except: a) A sample with hyponatremia and an elevated plasma osmolality should be suspected as

having pseudohyponatremia. b) SIADH is the most common cause of hyponatremia in hospital patients. c) Causes of SIADH include hypothalamic disorders, pulmonary diseases, ectopic ADH

production by tumors, various drugs and other causes.

Page 79: CP RIse Review

Chemistry Review Questions ⎯ Kent Lewandrowski, M.D. and Anand Dighe M.D., Ph.D.

Page 5 of 13

d) Explanations for pseudohyperkalemia include hemolysis, leukocytosis, and an elevated platelet count.

e) In patients with hypokalemia, a urine potassium less than 20 indicates a renal etiology. 23. All of the following concerning thyroid function testing are true except: a) Reverse T3 is an inactive hormone. b) The most common cause for an elevated TBG is pregnancy or estrogen administration. c) Serum TSH is the single best screening test to evaluate thyroid function. d) The T3 resin uptake (THBR) is a direct measure of the free thyroid hormone level. e) Thyroglobulin is principally useful as a tumor marker to monitor patients with known thyroid

carcinoma. 24. All of the following are true except: a) The most common causes of hyperthyroidism include Grave’s disease, toxic multinodular

goiter, and toxic adenoma. b) In patients with hyperthyroidism, the TSH is usually increased. c) The most common cause of hypothyroidism is Hashimoto’s thyroiditis. d) In most cases of hypothyroidism, the T3 resin uptake is low. e) Typical features of euthyroid sick syndrome include low total thyroid hormone levels, an

increased reverse T3, an increased T3 resin uptake, and either normal or slightly abnormal free hormones and TSH.

25. All of the following are true concerning Cushing’s Syndrome except: a) The diagnosis of Cushing’s Syndrome requires documentation of excessive cortisol

production and resistance to suppression (or loss of the diurnal variation). b) The overnight 1 mg DMS suppression test is the best initial screening test for Cushing

Syndrome. c) The cosyntropin test is useful in the confirmation of Cushing Syndrome. d) Normal subjects will suppress cortisol levels in response to DMS. e) The ACTH level in patients with pituitary Cushing’s disease may be normal, but inappropriate

for the level of hypercortisolism. 26. All of the following are true except: a) Patients with primary hyperaldosteronism typically exhibit hypertension and hypokalemia. b) Patients with primary hypoaldosteronism typically exhibit hypertension and hypokalemia. c) The saline suppression test using oral sodium chloride or saline infusion is useful in the

evaluation of primary hyperaldosteronism. d) Characteristic findings in patients with 21-hydroxylase deficiency include an elevated 17 OHP

and elevated urinary pregnane triol. e) Patients with 11-beta hydroxylase deficiency exhibit an elevated 11 deoxycortisol. 27. All of the following are true except: a) The five hour oral glucose tolerance test is useful for the diagnosis of reactive hypoglycemia. b) The diagnosis of diabetes mellitus type 1 may include demonstration of a fasting plasma

glucose greater than or equal to 125 mg/deciliter on more than one occasion. c) Measurement of urine microalbumin is useful to detect early treatable renal disease in

patients with diabetes mellitus. d) Criteria for hypoglycemia (Whipple’s triad) include presence of typical symptoms,

documented hypoglycemia and relief of symptoms by glucose administration. e) In patients with insulinoma and a normal serum insulin level, the insulin to glucose ratio may

be abnormal reflecting inappropriate hyperinsulinism.

Page 80: CP RIse Review

Chemistry Review Questions ⎯ Kent Lewandrowski, M.D. and Anand Dighe M.D., Ph.D.

Page 6 of 13

28. All of the following are true except: a) Common methods for measurement of the total protein include dye binding and biuret

methods. b) Hyperalbuminemia almost always reflects dehydration. c) Negative acute phase reactants include albumin and fibrinogen. d) The most common methods for albumin measurement are based on dye binding. e) Immunochemical methods for albumin measurements are rarely utilized. 29. Concerning Alpha 1 antitrypsin deficiency, the following statements are true except: a) The PiMM is the most common phenotype in the US population. b) The PIZ allele is the most common abnormal allele. c) The heterozyogote frequency of PiZM is less than 1%. d) Phenotyping of Alpha 1 antitrypsin is useful to evaluate an abnormal AAT level. e) Alpha 1 antitrypsin is an acute phase reactant. 30. All of the following statements are true except: a) Patients with Wilson’s disease exhibit a low serum ceruloplasmin. b) Patients with Wilson’s disease exhibit a low plasma copper level. c) Patients with Wilson’s disease exhibit an elevated urine copper level. d) Liver biopsy in patients with Wilson’s disease may show elevated copper, but this finding is

not definitive. e) The defect in Wilson’s disease results from an inborn error in ceruloplasmin. 31. All of the following are true except: a) The most common laboratory methods for measurement of catecholamines are based on

colorimetric assays. b) The clonodine suppression test is useful to evaluate patients with borderline catecholamine

levels. c) Urine metanephrines and more recently plasma metanephrines are the best screening tests

for pheochromocytoma. d) In patients with neuroblastoma, the urine VMA and/or HVA are elevated in most cases. e) Platelet serotonin levels and 5-HIAA are most useful to evaluate patients for carcinoid

syndrome. 32. All of the following are true except: a) DHEAS levels reflect adrenal androgen production. b) Urine 17 ketosteroids predominantly reflects testosterone metabolites. c) In a patient with primary hypogonadism, the LH and FSH will be elevated. d) The most common cause of adult male gynecomastia is due to drug associated causes. e) Prolactinoma is the most common pituitary adenoma. f) The most useful test for the diagnosis of acromegaly is the serum IGF-I. 33. TRUE or FALSE. A patient exhibits a total cholesterol of 180, an LDL cholesterol of 90, and

an HDL cholesterol of 65. Collectively, this reflects a favorable lipid profile using current classification systems.

34) All of the following are true regarding the volume of distribution for a drug EXCEPT: a) Drugs that are poorly water-soluble have low volumes of distribution. b) Drugs that are highly plasma protein bound tend to have lower volumes of distribution c) Many of the drugs of abuse (heroin, cocaine) have high volumes of distribution. d) The volume of distribution can be greater than the total body water of an individual.

Page 81: CP RIse Review

Chemistry Review Questions ⎯ Kent Lewandrowski, M.D. and Anand Dighe M.D., Ph.D.

Page 7 of 13

35) All of the following drugs circulate bound to albumin EXCEPT: a) Phenytoin b) Valproic acid c) Lidocaine d) Carbamazepine 36) All of the following factors may improve HPLC resolution EXCEPT: a) Increasing the column length b) Increasing the flow rate c) Using smaller beads to pack the column d) Changing the polarity of the solvent used for the mobile phase 37) In “reversed phase” HPLC which ONE of the following is correct: a) The stationary phase (column) is more polar than the mobile phase b) Reversed phase HPLC can only give qualitative results c) Drugs that are the most polar will elute last d) Drug metabolites generally elute prior to the parent drug 38) All of the following are true of ethylene glycol toxicity EXCEPT: a) Ethylene glycol can be measured by gas chromatography b) Ethylene glycol toxicity is associated with an increased anion gap and a normal osmolar gap c) Calcium oxalate crystals may be present in the urine d) Serum calcium levels may be decreased 39) In urine toxicology which ONE of the following compounds is most suggestive of heroin use? a) Morphine 6-0-glucuronide b) Morphine c) 6-mono-acetyl morphine d) Morphine 3-0-glucuronide 40) All of the following analytes may be increased by prolonged standing during phlebotomy EXCEPT: a) Platelets b) Cholesterol c) Calcium d) Sodium 41) All of the following analytes show high levels of intraindividual (biologic) variation EXCEPT: a) AST b) Albumin c) Creatine kinase d) Cortisol 42) All of the following are pre-analytic causes of a high serum potassium level EXCEPT: a) Thrombocytopenia b) Long tourniquet application times c) Excessive fist clenching during phlebotomy d) Traumatic draw

Page 82: CP RIse Review

Chemistry Review Questions ⎯ Kent Lewandrowski, M.D. and Anand Dighe M.D., Ph.D.

Page 8 of 13

43) Drawing of an additive potassium EDTA tube prior to a non-additive tube may result in all of the following EXCEPT: a) Decreased calcium levels in the non-additive tube b) Decreased alkaline phosphatase levels in the non-additive tube c) Increased magnesium levels in the non-additive tube d) Increased potassium levels in the non-additive tube 44) Excessive exposure to light is known to decrease all of the following analytes EXCEPT: a) Vitamin A b) Vitamin D c) Vitamin E d) Bilirubin 45) Exercise prior to phlebotomy may lead to increases in all of the following EXCEPT: a) Uric acid b) AST c) Potassium d) Creatine kinase 46) The half life of albumin is approximately: a) 6 days b) 12 days c) 18 days d) 6 weeks 47) All of the following are acute phase reactants EXCEPT: a) C3 b) Transferrin c) Alpha-1 anti-trypsin d) Alpha-2 macroglobulin 48) All of the following are true of patients with macroamylasemia EXCEPT? a) Patients have a circulating antibody directed against amylase b) In macroamylasemia the amylase clearance ratio is < 1% c) High levels of urinary amylase are associated with macroamylasemia d) Macroamylasemia may be present in up to 1% of the normal population 49) Which ONE of the following is true of serum amylase measurement? a) Amylase elevation is a highly specific marker of acute pancreatitis b) A high level of amylase is predictive of poor prognosis in acute pancreatitis c) Amylase remains elevated for 2-3 weeks following an episode of acute pancreatitis d) Serum amylase may be elevated in renal failure in the absence of pancreatitis 50) Which of the following conditions is associated with elevation of S-type (salivary) amylase: a) Renal failure b) Pancreatic pseudocysts c) Ectopic pregnancy d) Macroamylasemia

Page 83: CP RIse Review

Chemistry Review Questions ⎯ Kent Lewandrowski, M.D. and Anand Dighe M.D., Ph.D.

Page 9 of 13

51) A drug is said to show zero order elimination. This means: a) A constant fraction of the drug will be eliminated per unit of time b) A constant amount of the drug will be eliminated per unit time c) This drug will have a low volume of distribution d) The drug will have a high volume of distribution 52) The isoform of the cytochrome p450 system responsible for the increased sensitivity of patients to Warfarin is: a) CYP3A4 b) CYP2D6 c) CYP2C9 d) CYP2E1 53) The isoform of the cytochrome p450 system responsible for over 50% of phase I biotransformation reactions is: a) CYP3A4 b) CYP2D6 c) CYP2C9 d) CYP2E1 54) The following are all causes of elevated ammonia levels EXCEPT: a) Delays in sample processing b) Icing samples prior to testing c) Hemolysis d) Urea cycle disorders 55) The principle governing spectrophotometry (Beer’s law) is expressed by which ONE of the following equations, where A = absorbance a = molecular absorptivity b = cuvet path length C = analyte concentration a) A = a*b*C b) A = (a*c)/b c) A = (a*b)/C d) A = (b*c)/a 56) All of the following are causes of an elevated alkaline phosphatase EXCEPT? a) Pregnancy b) Paget’s disease c) Primary biliary cirrhosis d) Hypoparathyroidism 57) The following are true of urine immunoassays for cannabinoids EXCEPT: a) Cannabinoids are detectable in the urine up to 4 hours after a single use b) The assays commonly detect inactive metabolites such as 11-Nor-Δ9-THC-9-carboxylic acid c) False positive are rare

Page 84: CP RIse Review

Chemistry Review Questions ⎯ Kent Lewandrowski, M.D. and Anand Dighe M.D., Ph.D.

Page 10 of 13

d) Chronic cannabinoid use may markedly increase the detection period in the urine 58) All of the following are true of urine drugs of abuse immunoassays EXCEPT: a) False positives may occur in the phencyclidine immunoassay due to dextromethorphan b) False positives may occur in the amphetamine immunoassay due to cold medications (pseudoephedrine) c) Poppy seed ingestion may cause false positives in the opiate immunoassay d) Naloxone (opiate antagonist) may cause false negative opiate immunoassay results 59) Ion selective electrodes measure electrolytes in a reaction that is defined by: a) Beer’s law b) The Nernst equation c) Graham’s law d) The Rydberg equation 60) All of the following analytes may be measured by atomic absorption spectroscopy EXCEPT: a) Lead b) Aluminum c) Selenium d) Lithium 61) All of the following analytes may be measured by atomic emission (flame photometry) EXCEPT: a) Lithium b) Sodium c) Potassium d) Zinc 62) Numerous enzymatic assays (e.g. cholesterol) in the laboratory result in the formation of hydrogen peroxide. Why is this a useful reaction product? a) Hydrogen peroxide can cause enzymatic color change of a chromagen, which can then be measured b) Hydrogen peroxide has a unique absorbance spectrum c) Hydrogen peroxide can be readily converted to water d) Hydrogen peroxide forms superoxide anions that can then be readily detected 63) The ion selective electrode for potassium is commonly: a) Calomel and glass b) Platinum c) Valinomycin d) None of the above. 64) Which ONE of the following compounds absorbs strongly at 340 nm: a) NAD+

b) NADH c) Bilirubin d) Hemoglobin 65) All of the following statements about spectrophotometry are true EXCEPT:

Page 85: CP RIse Review

Chemistry Review Questions ⎯ Kent Lewandrowski, M.D. and Anand Dighe M.D., Ph.D.

Page 11 of 13

a) Spectrophotometer wavelength accuracy may be verified with holmium oxide or didymium b) Spectrophotometer system accuracy may be verified with potassium chromate c) High level light transmission is associated with analytes with low levels of absorbance d) Diffraction gratings are used to detect light of specific wavelengths 66) Which ONE of the following is NOT an international system (SI) unit? a) Mole b) Candela c) Gram d) Meter 67) The concept underlying osmolality measurement is: a) Freezing point elevation by increased solute particles b) Freezing point depression by increased solute particles c) Boiling point elevation by increased solute particles d) Boiling point depression by increased solute particles 68) In spectrophotometry, the relationship between the percent transmittance and absorption is expressed by which ONE of the following equations: a) Abs = 2 – log (%Transmittance) b) Abs = 2 + log (%Transmittance) c) Abs = 2 – 1/(log (%Transmittance)) d) Abs = 2 + 1/(log (%Transmittance)) 69) All of the following are true of homogeneous and heterogeneous enzyme immunoassays EXCEPT: a) Homogeneous assays require a separation step b) In homogenous assays the labeled reactant behaves differently when bound c) In heterogeneous assays the physical properties of the label are not changed in the binding

process d) Fluorescence polarization immunoassays are an example an homogeneous assay. 70) An increase in the concentration of the binding protein for a drug generally results in:

a) Decreased total drug levels and decreased free drug levels b) Increased total drug levels and no effect on free drug levels c) Increased total drug levels and decreased free drug levels d) Decreased total drug levels and increased free drug levels 71) Assuming 100% bioavailability, the formula for the peak blood concentration of a drug for a drug is which ONE of the following: a) Peak concentration = dose/(volume of distribution) b) Peak concentration = dose x (volume of distribution) c) Peak concentration = (volume of distribution)/dose d) Peak concentration = 1/((dose) x (volume of distribution)) 72) All of the following are true concerning nephelometry EXCEPT: a) The technique is based upon light scatter b) Most applications using nephelometry measure rate of appearance of complex (rate nephelometry) to improve accuracy c) The detector is positioned at an angle to the light source

Page 86: CP RIse Review

Chemistry Review Questions ⎯ Kent Lewandrowski, M.D. and Anand Dighe M.D., Ph.D.

Page 12 of 13

d) It is the technique of choice for the measurement of proteins in the micromolar range 73) A gas chromatograph separates compounds according to their: a) Size b) Polarity c) Volatility d) Ionic charge 74) A child presents in the emergency room with a history of organophosphate insecticide poisoning. All of the following are true regarding toxicology testing for organophosphate poisoning EXCEPT: a) Pseudocholinesterase is reduced in patients with liver disease and may be increased in patients in an acute phase response. b) RBC cholinesterase levels are decreased in patients with shortened RBC lifespans (e.g. hemolysis) c) Organophosphate poisoning decreases both pseudocholinesterase and RBC cholinesterase. d) The definitive antidote for organophosphate poisoning is 2-PAM (pralidoxime). 75) In thin layer chromatography plate shown below, what is the retention factor (Rf) of compound X? Solvent front = 0.90

Origin

Compound X =0.60 a) 0.6 b) 0.6/0.9 = 0.67 c) (0.9-0.6)/0.9 = 0.33 d) 0.9-0.6 = 0.30 Answer Key: 1. D 2. C 3. D 4. C 5. D 6. A 7. B 8. E 9. A 10. A

11. E 12. D 13. C 14. B 15. A 16. B 17. A 18. D 19. B 20. A

21. E 22. E 23. D 24. B 25. C 26. B 27. A 28. C 29. C 30. E

31. A 32. B 33. T 34. A 35. C 36. B 37. D 38. B 39. C 40. D

Page 87: CP RIse Review

Chemistry Review Questions ⎯ Kent Lewandrowski, M.D. and Anand Dighe M.D., Ph.D.

Page 13 of 13

41. B 42. A 43. C 44. B 45. C 46. C 47. B 48. C 49. D 50. C

51. B 52. C 53. A 54. B 55. A 56. D 57. A 58. D 59. B 60. D

61. D 62. A 63. C 64. B 65. D 66. C 67. B 68. A 69. A 70. C

71. A 72. D 73. C 74. B 75. B

Page 88: CP RIse Review
Page 89: CP RIse Review

Management Section 1. Which of the following is not an operating expense?

a. Wages b. Benefits c. Equipment maintenance d. Equipment purchase e. Telephone charges

2. Which of the following expenses maintain production, but are not directly associated with the

product? a. Direct cost b. Indirect cost c. Variable cost d. Fixed cost e. Capital cost

3. Which of the following is not a fixed cost?

a. Rent b. Marketing c. Management salaries d. Depreciation e. Office supplies

4. Cost accounting is a tool to:

a. Justify a new technologist position b. Track expenses c. Analyze the budget variances d. Justify a capital request e. Calculate salaries

5. Which analysis might be used to determine if a test should be brought in-house?

a. Net present value b. Break even c. Pay back d. Benchmark e. Variance

6. According to McGregor:

a. Esteem, ego, and status are the highest human needs b. Managers can motivate employees by using satisfiers such as responsibility, acheivement

and advancement. c. Company X managers assume employees naturally strive for excellence d. Company Y managers assume employees naturally strive for excellence e. Company X managers rely on union leadership to run the operation

7. According to Maslow’s heirarchy:

a. Esteem, ego, and status are the highest human needs b. Social needs are the most basic human need c. Self-actualization and fulfillment are the highest human needs d. Managers should address one human need at a time e. Entry level employees do not have higher human needs

Page 90: CP RIse Review

8. Based on current labor law:

a. An employer must provide complete accommodations for handicapped employees b. All positions in an organization must have a job description c. Healthcare facilities are not bound by the National Labor Relations Act d. An employer must offer health insurance to all employees e. A job candidate is not required to state their marital status

9. The Medicare program

a. Provides healthcare coverage for anyone who wants to pay for it b. Determines the reimbursement rate for all health insurance payors c. Establishes eligibility criteria for individuals and providers d. Covers mostly children e. Will pay for any medical service provided

10. Medicare will reimburse laboratory services from a fee schedule for:

a. Outpatient lab tests b. Inpatient lab tests c. Outpatient and inpatient lab tests d. All tests, but only in certain states e. All tests sent to a reference laboratory

11. Medicare Part A:

a. Covers inpatient services, home health, physician profession services b. Is administered by a fiscal intermediary hired by Health and Human Services c. Is funded by each state d. Covers outpatient visits e. Includes HMO options

12. Medicare reimbursement for outpatient lab services

a. Is based on a national fee schedule b. Are included in the Ambulatory Payment Classification (APC) c. Is based on the laboratory charges d. Is based on the local fee schedule e. Covers all screening tests

13. An Advance Beneficiary Notice is required:

a. For all outpatient laboratory services b. For all Medicare outpatient laboratory services c. For Medicare outpatient laboratory charges that may be denied d. If it is likely that the payment will be denied e. Only for screening tests

14. Which of the following is not a common HMO model?

a. Individual practice association b. Fee for service c. Group model d. Staff model e. Mixed model

Page 91: CP RIse Review

15. All organizations performing laboratory tests are required to obtain:

a. CAP inspections b. OSHA license c. CLIA license d. FDA approval e. JCAHO inspection

16. The goal of CLIA88 is to:

a. Reduce fraud and abuse b. Ensure the safety of laboratory workers c. Document medical necessity d. Improve quality and reduce errors e. Improvement reimbursement for laboratory tests

17. Which of the following is not a requirement in CLIA88 regulations?

a. Quality control b. Personnel c. Reimbursement d. Proficiency tests e. Inspections

18. The following organizations are CLIA approved for accreditation

a. College of American Pathologists b. Joint Commission for the Accreditation of Healthcare Organizations c. Food and Drug Administration d. A and B e. B and C

Page 92: CP RIse Review
Page 93: CP RIse Review

Answers:  Laboratory Administration – Donna MacMillan, MT(ASCP), MBA  

1. D 

2. B 

3. E 

4. B 

5. B 

6. D 

7. C 

8. E 

9. C 

10. A 

11. A 

12. D 

13. C 

14. B 

15. C 

16. D 

17. C 

18. D 

 

Page 94: CP RIse Review
Page 95: CP RIse Review

ASCP RESIDENTS’ REVIEW COURSE 2006 PATHOLOGY INFORMATICS

Walter H. Henricks, M.D. REVIEW QUESTIONS 1. Regarding bytes in computer systems, all of the following statements are true except:

A. A byte consists of a string of eight bits B. A byte is the data unit typically used to represent characters such as letters C. Computer storage is typically measured in multiples of bytes D. The abbreviation for byte is “B” E. A byte has a value of 1 or 0 in the binary number system

2. Which of the sequences below correctly ranks the data capacity of data storage devices from lowest to highest?

A. floppy disk drive < CD-RW < 30 GB hard disk drive < DVD B. floppy disk drive < CD-RW < DVD < 30GB hard disk drive C. floppy disk drive < DVD < CD-RW < 30 GB hard disk drive D. CD-RW < floppy disk drive < DVD < 30 GB hard disk drive E. floppy disk drive < 30 GB hard disk drive < CD-RW < DVD

3. Which of the following statements best characterizes what happens to data in RAM and in hard disk storage when electrical power to the computer system is interrupted or the system crashes?

A. All data in RAM and on disk are transferred to the CPU for temporary holding.d

B. Data in RAM are retained, and data on disk are lost C. Data in RAM are lost, and data on disk are retained D. Data in RAM and data on disk are both retained E. Data in RAM and data on disk are both lost

4. In a client/server computer system:

A. networking technology is not required B. clients are the devices that initiate requests and processes C. all of the computing functions are handled on the main computer D. systems management is simplified compared to a mainframe-based system E. a graphical user interface is typically not used

Page 96: CP RIse Review

5. Which of the following telecommunications technologies for connecting to remote sites offers the fastest rate of data transmission?

A. Standard ISDN B. Fractional T1 C. Regular telephone service D. Leased line capable of 56 kbps E. T1

6. The level of encryption generally considered to be strong encryption for data transmission is:

A. 32-bit B. 40-bit C. 56-bit D. 64-bit E. 128-bit

7. The process of verifying identity of a user logging in to an LIS is most specifically referred to as:

A. encryption B. confidentiality C. security D. authentication E. authorization

8. The standard computer language used to query relational databases is:

A. HTML B. SQL C. HL7 D. LOINC E. ASTM

9. In a relational database:

A. data tables are related by shared common data fields B. redundant records are typically found in a given table C. the first record in a table is known as the primary key D. each table is a single record E. different tables often have a majority of data fields in common

Page 97: CP RIse Review

10. All of the following might be found in an LIS dictionary except: A. reference ranges for test results B. test name mnemonic/synonym C. results of individual patients’ tests D. physician names E. special stain types

11. Autoverification in an LIS:

A. is the automatic transfer of information from the LIS to the electronic medical record

B. forbidden by law C. is the process of signing out reports electronically by use of an electronic

signature D. is release of results from automated analyzers to an LIS without prior human

review E. requires a manual review before final release of test results

12. HL7:

A. is implemented in relatively few interfaces between information systems in healthcare

B. specifies the rules and syntax for electronic exchange of healthcare data C. is most often implemented in interfaces between automated analyzers and an

LIS D. used only in laboratories E. eliminates the need for interface engines

13. A formal agreement under which a client may obtain software source code in the event of vendor insolvency is known as a(an):

A. request for proposal B. application service provider agreement C. escrow agreement D. service level agreement E. gentleman’s agreement

14. According to law, anatomic pathology reports, or exact duplicates, must be retained for at least:

A. 1 year B. 2 years C. 5 years D. 10 years E. 20 years

Page 98: CP RIse Review

15. Select from the following statements the one that best characterizes the use and disclosure of protected health information (PHI) under HIPAA:

A. Searches of the LIS results database for quality assurance purposes are not a permitted use of PHI.

B. A reference laboratory must get consent from its clients’ patients to see the PHI of such patients when performing consultative pathology work on behalf of the patient’s treating physician.

C. A laboratory’s LIS software vendor must sign an agreement stipulating that the vendor will protect PHI to which the vendor has access at the client site during the course of providing services.

D. An Institutional Review Board (IRB) may not grant an exception to the requirement for patient’s authorization to use his/her PHI for a research project.

E. Under no circumstances is a laboratory permitted to transmit test results using fax machines

16. The used digital image file format used for display of digital photographs, such as photomicrographs, on websites is:

A. GIF B. JPEG C. TIFF D. Bitmap E. PICT

Page 99: CP RIse Review

PATHOLOGY INFORMATICS Walter H. Henricks, M.D.

Answers:

1. E

2. B

3. C

4. B

5. E

6. E

7. D

8. B

9. A

10. C

11. D

12. B

13. C

14. D

15. C

16. B

Page 100: CP RIse Review
Page 101: CP RIse Review

COLOR PAGE